Diskussion:Bellsche Ungleichung/Archiv

aus Wikipedia, der freien Enzyklopädie
Letzter Kommentar: vor 6 Monaten von Haraldmmueller in Abschnitt Ungrammatischer Satz?
Zur Navigation springen Zur Suche springen

Ordentlicher Artikel

Das ist doch mal ein ordentlicher Artikel!--CWitte 14:25, 28. Okt 2004 (CEST)

Hm, der Absatz Weil verschiedene Experimente diese Beziehung verletzen, wird von der Mehrzahl der Quantenphysiker gefolgert, dass diese Annahmen, im besonderen Lokalität und Realismus, unvereinbar sind und in einer konsistenten Theorie nicht beide erfüllt sein können. Insbesondere lässt sich unmittelbar überprüfen, dass die Ungleichung in der mathematischen Formulierung der Quantenmechanik klar verletzt wird (siehe hierzu auch das Kochen-Specker-Theorem, ein anderes No-Go-Theorem über realistische Modelle). verwirrt mich.

Die mir vorher bekannte Folgerung aus der Bell'schen Ungleichung war, dass es keine versteckten lokalen Variablen gibt, die das Ergebnis voherbestimmen (und Einstein's "der Alte würfelt nicht" falsch ist) – und das (die Kopenhagener Interpretation) kommt weiter unten auch als mögliche Folgerung. Ich dachte bisher, dass mit "Realismus" der QM gerade gemeint ist, dass die QM-Beschreibung der Welt vollständig ist, es also keine "hidden parameter" gibt. Hier steht jetzt aber, dass die Experimente zeigen, dass Lokalität und Realismus nicht beide erfüllt sein können. Wenn mich als theoretischen Physiker schon verwirrt, was soll dann ein Laie erst sagen? --Christian Gawron 22:59, 25. Dez 2005 (CET)

Übersicht

Es wäre schön, wenn jemand genauer erklären könnte, was mit den Begriffen Lokalität, Realität und gemeinsame Messbarkeit in diesem Zusammenhang gemeint ist (und am besten auch, wie diese Begriffe mit verborgenen Variablen und der Kopenhagener Interpretation zusammenhängen). --Christian Gawron 15:29, 3. Jan 2006 (CET)


Hab die Uebersicht ueberarbeitet, auch der Rest vom Artikel ist noch nicht perfekt (Lokalitaet und "keine Signaluebertragung" sind nicht genau dasselbe! etc) werde bald die anderen Sachen noch ueberarbeiten. --Weissenburg 23:45, 17. Jun 2006 (CEST)

Ein paar Anmerkungen auf die Schnelle:

  • Üblicher ist mE der Plural (Bellsche Ungleichungen)
  • "freier Wille", "Realismus" in dieser Form einzuführen weicht vom EPR-Text ab und ist kontrovers
  • ebenso die Schlussfolgerung bzgl "nichtlokal-realistischen" Theorien
  • warum werden zwei allerweltseinführungen (zeilinger, rae, außerdem der sakurai) zur qm überhaupt aufgelistet (als ob epr+bell nicht in praktisch jedem textbook vorkäme) anstatt einschlägige fachliteratur? (habe ein paar wichtige titel ergänzt)

Ca$e 23:57, 28. Aug 2006 (CEST)

"Realismus bedeutet, dass das Gesamtsystem bereits die vollständige Information über mögliche Wahrscheinlichkeiten von Messergebnissen enthält, also dass es eine beobachterunabhängige Realität gibt" Nach den Erkennntnissen der heutigen Physik gibt es genau das, nämlich eine beobachterunabhängige Realität, nicht bzw. ist sie nicht beobachtbar. Der Zustand von Systemen (Teilchen) ist solange unbestimmt bis eine Messung daran durchgeführt wird.

Zitat Prof.Anton Zeilinger:

"Letztlich ist es so, dass wir in der Quantenphysik über die Wirklichkeit an sich keine direkten Aussagen machen können. Wir können nur darüber reden, welche Information, welches Wissen wir haben. Wir können uns eine Wirklichkeit dann konstruieren, wenn mehrere Physiker in der gleichen Situation die gleichen Eindrücke, die gleichen Informationen haben. Dann werden wir sagen, dass dies wahrscheinlich der Wirklichkeit entspricht.

Das heißt, es kommt auf die intersubjektive Übereinkunft an, die aber nicht alles ist. Denn es ist offenbar doch so, dass es dahinter eine Wirklichkeit gibt. Bloß über die Wirklichkeit selbst etwas definitiv auszusagen, ist nicht möglich." Quelle: http://www.philosophische-praxis.at/zeilinger.html (Interview Wiener Zeitung)

Wahrscheinlichkeiten/Messfrequenzen

Die Berechnung der Bellschen Ungleichungen durch klassische Physik und Quantenmechanik gefällt mir eigentlich sehr gut, aber in einem Absatz der QM scheinen mir ein paar Fehler zu sein? Hier eine verbesserte Version:

---

Man betrachte P(c+,b+), wobei Alice in c-Richtung misst und mit der Wahrscheinlichkeit 1/2 den Wert +1/2 erhält. Dadurch kollabiert die Wellenfunktion zu |c-B. Im Zustandsraum von Bobs Elektron kann die bedingte Wahrscheinlichkeit berechnet werden, dass Bob bei der Messung in Richtung b +1/2 erhält (die Indizes für b sind weggelassen):

P(c+,b+) = 1/2 | b+|c- |2
= 1/2 | b+| D(y, -θ) |b- |2

---

Wichtig erscheint mir das 〈b+|c-〉. Unser Ausgangszustand ist ja gerade c- und wir projizieren auf b+. Auch sollte man vielleicht einmal die Wellenfunktion der Verschränkung einmal explizit hinschreiben, sonst versteht man das irgendwie nicht.

Traue mich nicht, dass einfach so zu ändern, vielleicht ist es ja falsch ;). Würde mich also über Rückmeldung freuen.

Der Artikel wuerfelt Frequenzen mit Wahrscheinlichkeiten durcheinander.

http://de.wikipedia.org/wiki/Bellsche_Ungleichung#Verborgene_Variablen

Die Tabelle besteht aus den Anzehlen der Messungen des jeweiligen Ergebnisses. Im Text werden Summen aus diesen Anzahlen dann allerdings als Wahrscheinlichkeiten bezeichnet. Dies ist so natuerlich nicht korrekt. Die Messfrequenzen gehen im stochastischen Limes in die Wahrscheinlichkeiten ueber, in dem Falle, dass man unendlich viele Messungen macht. FlorianPaulSchmidt 10:51, 10. Sep 2006 (CEST)

Klar, in einem deterministen Modell mit verborgenen Variablen wird gibt es eigentlich keine Wahrscheinlichkeiten, sondern die Teilchen gehören zu einer der Sorten 1 bis 8 und eine Messung ergibt das aus dieser Sorte vorbestimmte Ergebnis. In einer Messreihe würden die Häufigkeiten der verschiedenen Messergebnisse bestimmt, die sich aus der Anzahl der Teilchen der verschiedenen Sorten ergeben. Egal wie die Häufigkeiten der Sorten verteilt sind, gilt immer die Bellsche Ungleichung und zwar genau genommen für die gemessenen Häufigkeiten und nicht für Wahrscheinlichkeiten.

Geloeschter Satz

Folgenden Satz bzgl der Bohmschen Mechanik geloescht:

"Eine solche Interpretation wird jedoch als unelegant angesehen, da alle Teilchen des Universums instantan mit allen anderen Teilchen die Informationen austauschen können müssten."

Einen aehnliche Satz mit subjektiver Kritik koennte man an die jeweilig anderen Interpretation mit jeweils geaenderter Begruendung auch anhaengen. Zum Beispiel "...wird als unelegant angesehen, da eine Kollaps der Wellenfunktion waehrend dem Messprozess notwendig ist." oder "...wird als unelegant angesehen, da Viele-Welten Modelle mit fragwurdigem empirischen Gehalt notwenig sind." Wieso also ausgerechnet bei der Bohmschen Mechanik? --Nost 23:58, 7. Okt 2006 (CEST)

Lokal-realistisch oder deterministisch

Was unter dem Begriff lokal-realistisch zu verstehen sein soll, ist mir nicht wirklich klar geworden. Ich denke aber es geht im Wesentlichen um die Frage, ob das Messergebnis (durch verborgene Variablen) vorherbestimmt ist oder erst nach der Messung feststeht.

Die Bellsche Ungleichung ergibt sich schlicht aus der Annahme, dass die Messergebnisse von Bob und Alice nicht, wie die Quantenmechanik besagt, erst nach der Messung feststehen, sondern bereits zuvor durch nicht messbare Eigenschaften der Teilchen (verborgene Variablen) bestimmt oder determiniert sind. Unter dieser Voraussetzung spielt es gar keine Rolle wie diese Messung ausgeführt wird.

Es gibt vielmehr 8 verschiedene Möglichkeiten, welche Werte die relevanten vorborgenen Varibalen der von Bob detektierbaren Teilchen haben können. Die von Alice detektierbaren Teilchen sind damit ebenfalls in ihren Eigenschaften festgelegt. Wenn es von den 8 verschiedenen Teilchensorten jeweils eine bestimmte Anzahl N1,N2, ... bis N8 gibt, folgt daraus unmittelbar die Bellsche Ungleichung. Offenbar widerspricht sowohl das Experiment als auch die Quantenmechanik dieser Vorstellung einer deterministischen Welt. Ferner ist eine rein deterministische Welt kaum mit der Vorstellung eines freien Willens vereinbar, da letzlich auch die Vorgänge in unserem Gehirn, unsere Gedanken und Entscheidungen bereits vorbestimmt wären. Die Experimente zeigen zudem, dass die Gesetze der Quantenmechanik nicht nur in atomaren Dimensionen gültig sind. 84.169.204.242 21:25, 25. Dez. 2006 (CET)

Ich denke "lokal-realistisch" ist einfach nichts als der Einzelfall. Im Grunde ist Einsteins Gedanke, wenn ich ihn verstanden habe, der einfachste, nämlich die Quantenmechanik beschreibt nur Ensemble, keinen Einzelfall und ist darum prinzipiell unvollständig. Da man den Einzelfall aber schlecht leugnen kann erklärt man ihn für prinzipiell unerforschbar. Das heist man gibt die Unvollständigkeit zunächst zu und erklärt sie dann aber zum Naturgesetz. Eine "Freiheit des Willens" kann man weder hier noch dort rauslesen. Es ist aber interessant, daß genau das, also der freie Wille, auch das emotionale Hauptargument etwa von Born gegen Einstein war - eine restlos kausal determinierte Welt sei ihm unerträglich. Wobei ihm der Witz fehlte zu sehen, daß er den Sack nur eben von der anderen Seite zugezogen hat. Also in den Bellschen Gleichungen eine Art mathematischen Freiheitsbeweis zu sehen ist einfach abwegig, völlig unabhängig von der Frage, ob sie zu einer richtigen Interpretation des Einzelfalls beitragen oder nicht. - Der Artikel gefällt mir, nebenbei, weder in seinem Ton, noch in seiner verkrampften Umständlichkeit und Unanschaulichkeit. Nur schon weit Eingelesene können hier folgen und die bevorzugen wohl andere Quellen. --M.sack 11:41, 3. Mär. 2007 (CET)

Fehler in Herleitung

Mal so als Frage: Es sind doch drei Versuchsreihen in der klassischen Herleitung. Müsste demnach nicht N_4 bei p(a,b) eine andere Größe sein als bei p(a,c)? Das ist zwar im Original-Paper auch falsch, aber demnach müsste man ja die Reihen umsortieren, damit man so rechnen kann und damit ginge die Herleitung nur, wenn eine gemeinsame Wahrscheinlichkeitsverteilung unter den drei Versuchsreihen vorlegen kann (sodass man (a1,b1), (b2,c1), (a2,c2) zu (a,b), (b,c), (a,c) vereinfachen kann). Oder irre ich mich?

Soweit ich erkennen kann ist die Quelle für den speziellen Beweis der Ungleichung im Kapitel nicht angegeben. Ist jemandem die Quelle bekannt? T.S. 08:22, 7. Aug. 2008 (CEST)

Kritik an der Voraussetzung

Ein Punkt fehlt meineserachtens ... die Kritik an der Voraussetzng. Es geht bei dem Beweis um Wahrscheinlichkeiten, die jedoch nur unter der Voraussetzung relevant sind, daß eben die eine Grundvoraussetzung korrekt ist: der Quantenmechanische Zustand eines beobachteten Partikels ist bis zum Zeitpunkt der Beobachtung unbestimmt ... Die Quantenmechanik geht nicht nur davon aus, sondern basiert größtenteils auf genau dieser Annahme, die jedoch selbst nicht beweisbar ist ... Es wird davon ausgegangen, daß ein solches Partikel jederzeit jeden beiebigen Zustand annehmen kann und daß erst um Zeitpunkt der Messung dieser Zustand festgelegt ist, da damit das einzelne Partikel zerstört wird und somit kein nachfolgener abweichender Zustand mehr gemessen werden kann. Falls dem jedoch nicht so ist, falls also der Quantenzustand eines Partiels auf Grund der angesprochenen verborgenen Variablen durchaus zum Zeitpunkt seiner Entstehung fixiert wird, so muß man dem Partikel keine unendliche Anzahl verborgener Variablen mit auf den Weg geben, die irgndwann und irgendwo im Ziel auf ihn einwirken, sondern dann ist die Wahrscheinlichkeitsrechung auf den Zeitpunkt des Ursprungs dises Partikel beschränkt ... und ist danach nichtmehr relevant ... Die Bellsche Ungleichng wäre dann ebenfalls bedeutungslos, da auch sie auf der angesprochenen Grundannahme beruht ... Chiron McAnndra 13:44, 6. Jan. 2008 (CET)

Bell'sches Theorem

Guter Artikel, jedoch - unverständlich - wissenschaftlich. So könnte man vereinfacht deuten, dass keine Theorie der Physik-Realität, die mit der Quantenmechanik (Quantentheorie) kompatibel ist, davon ausgehen kann, das räumlich getrennte Ereignisse voneinander unabhängig sind. Das hieße: alles ist Eins.


Anmerkung: Vorsicht: Möglicher Fehler! "Und"-Wahrscheinlichkeiten sollten multipliziert werden!


Der Spiegel berichtet in einem Artikel von heute ueber QM und Bell'sches Theorem. Meine Frage: Bellsche Ungleichung und Bell'sches Theorem, ist damit das gleiche gemeint oder sind das zwei verschiedene/unabhaengige Dinge? Gruss --Lofor 12:44, 5. Okt. 2008 (CEST)

Ja. Um Deine Frage auch anderen zu beantworten, werde ich eine Weiterleitung anlegen. --Norbert Dragon 16:27, 5. Okt. 2008 (CEST)
Danke fuer die Auskunft und Weiterleitung :) Gruss --Lofor 17:33, 5. Okt. 2008 (CEST)

Vergleich mit der Quantenmechanik

In dem Abschnitt "Vergleich mit der Quantenmechanik" habe ich einen Fehler korrigiert: Zu der Drehung D(y,theta) um den Winkel theta gehört bei Spin-1/2-Teilchen wie dem Elektron der Operator exp(i theta/2 sigma_y) mit theta/2 und nicht theta. Letzteres wäre für Spin-1-Teichen wie etwa Photonen richtig. Ich habe diese Korrektur in den entsprechenden Zeilen durchgeführt. Die Schlussfolgerung, dass die Bellsche Ungleichung im Widerspruch zur Quantenmechanik steht, wird dadurch natürlich nicht berührt. Rudolf Dann, 24. März 2008 (falsch signierter Beitrag von 91.39.224.71 (Diskussion) 21:38, 24. Mär. 2008 (CET))

Fußnote

Ich habe die Fußnote

Die Subtilität des quantenmechanischen Messprozesses wird klar an folgendem Beispiel für das Resultat eines Experimentes an einem Doppelspaltsystem, mit den beiden komplementären Einzelspalt-Eigenschaften A oder B. Die quantenmechanische Wellenfunktion, eine Wahrscheinlichkeitsamplitude, sei etwa , d.h. es wird nur die Aussage gemacht, dass die Ergebnisse komplementär sind, mit je 50% Wahrscheinlichkeit für einen der beiden Fälle, AB oder BA, und mit Singulett-Symmetrie (gegeben durch das negative Vorzeichen vor dem zweiten Term). Das Ergebnis der Messung selbst könnte BA sein, aber ebensogut AB. Die Symmetrie des Systems (bei positivem Vorzeichen vor dem zweiten Term spricht man von Triplett-Symmetrie) ergibt sich aus dem Interferenzterm der eigentlichen Wahrscheinlichkeit,

aus dem Artikel entfernt, weil sie meiner Ansicht nach unverständlich und falsch ist. Ein Doppelspalt-Zustand ist eine Superposition a A + b B, Zustände wie AB - BA hingegen bezeichnen Produktzustände (typischerweise zweier Spins). Was aber soll die Fußnote besagen? --Norbert Dragon 16:15, 14. Okt. 2008 (CEST)

Interessanterweise ergibt sich mit der Spin-Fassung der Fußnote einerseits ein konkretes Beispiel für Ihre Messprozess-Charakterisierung, "... feststellt, was zuvor nicht feststand" (nämlich Eigenschaft von Spalt (1)=A, verbunden mit ... (2)=B, und nicht etwa ... (1)=B verbunden mit ... (2)=A), die ich für bemerkenswert halte (in der Wellenfunktion, die ja "präpariert" wird, steht nur, dass die Messergebnisse konträr sein müssen, d.h. dass neben AB auch BA gleich wahrscheinlich ist, während z.B. AA oder BB nicht in Frage kommen. Also ist die Messung mehr als nur Präparation, nämlich, wie Sie sagen, Feststellung dessen, was zuvor nicht feststand, und andererseits (in der Bohmschen Fassung) genau das Beispiel, das Einstein für bedenklich hielt. Im Übrigen habe ich angenommen, dass der Singulett-Zustand eines Zwei-Spin-Systems, bekannt ist und dass man analog auch ein Doppelspalt-System beschreiben kann (A und B sollen und entsprechen, also das Produkt AB (eigentlich Tensorprodukt von Operatoren) soll gerade entsprechen.)
Das Beispiel beleuchtet nicht die Aussage, dass eine Messung feststellt, was früher nicht feststand, sondern einen viel spezielleren Sachverhalt bei Messung an verschränkten Paaren, bei denen Meßergebnisse korreliert sind. Beim Beispiel muß mehr erklärt werden als es erklärt. Insbesondere kann nicht vorausgesetzt werden, daß der Leser die Kurzschrift von Produktzuständen kennt. --Norbert Dragon 09:51, 15. Okt. 2008 (CEST)

Literaturnachtrag

Hallo, ich habe heute einige Literaturquellen nachgetragen auf die im Artikel explizit Bezug genommen wird ([1]). Da ich Fachfremd bin bitte ich die Hauptautoren die Richtigkeit der Angaben zu prüfen. Danke. --Cepheiden 14:07, 22. Okt. 2008 (CEST)

Ich habe mir den Artikel noch einmal durchgelesen. Er ist fast unlesbar und nicht für wikipedia-Leser als weiterführende Literatur geeignet. Ich habe ihn daher auskommentiert. --Norbert Dragon 17:16, 22. Okt. 2008 (CEST)
Es ging sicher um den Norsen2006-Artikel, den hab ich nicht eingefügt sondern nur die Quellenzitierung formatiert :-) Grüße --Cepheiden 17:31, 22. Okt. 2008 (CEST)
Ich habe das auch so verstanden. Schade um die wohl leider vergebliche Arbeit. Aber dennoch: Dankeschön. --Norbert Dragon 17:33, 22. Okt. 2008 (CEST)

"Freier Wille" als Prämisse für Bellsche Ungleichungen??

Im Artikel steht, dass "freier Wille" eine Voraussetzung für die Gültigkeit der Bellschen Ungleichung sei. In Bells Original-Artikeln und Review-Artikeln, wie z.B. dem SEP-Artikel], steht davon nichts.

@Benutzer:Weissenburg: Du nennst diesen Artikel als Beleg. Vermutlich meinst Du diesen Satz (Seite 3):

"It is essential that each observer can choose his/her test independently of what the other distant observers do (or have done, or will do)."

Diese Prämisse ist nicht äquivalent zur Annahme freien Willens. In realen Experimenten wird die "independent choice of what other distant observers do" durch automatisierte zufällige Einstellungen der Orientierung der Polarisatoren gewährleistet. Bei einer solchen Messanordnung von "freiem Willen" zu sprechen, ist m.E. weit hergeholt. Mit dem Begriff des freien Willens wird ein anthropozentrisches Element in das Thema hineingetragen, welches IMHO in der Fachliteratur nicht vorhanden ist. Die o.g. Quelle gibt dies jedenfalls nicht her. Wenn es keinen Beleg gibt, in dem klar und eindeutig postuliert wird, dass die Bellsche Ungleichung menschlichen freien Willen erfordert, sollte der Begriff aus dem Artikel entfernt werden.--Belsazar 14:50, 21. Okt. 2008 (CEST)

Ich verstehe die Bedenken gegen Formulierungen, die man als Einwirkung von Bewußtsein auf physikalische Messungen mißdeuten könnte. Genau genommen geht es nicht um den freien Willen des Beobachters, sondern um die Möglichkeit, zufällig auszuwählen, was gemessen wird. Das hinzugefügte zufällig sollte den Bedenken gerecht werden. Übrigens mißt man freien Willen an der Unmöglichkeit, das Handeln des Freien vorherzusagen. --Norbert Dragon 16:08, 21. Okt. 2008 (CEST)
  • In modernen Bell-Test-Experimenten, wie z.B. dem Experiment von Weihs et al., erfolgt die Einstellung der Polarisatorwinkel von Alice und Bob durch Zufallsgeneratoren, d.h. ohne menschliches Zutun. Inwiefern soll sich hier ein Zusammenhang zum freien Willen ergeben?
  • Die Formulierungen im Artikel lassen sich so deuten, dass der lokale Realismus aufrechterhalten werden kann, wenn die Prämisse des freien Willens fallengelassen wird. Mir ist keine einschlägige Veröffentlichung bekannt, die diese Option in Erwägung zieht.
Fazit: Wenn im Artikel etwas über "freien Willen" stehen soll, muss das durch Verweis auf seriöse Untersuchungen belegt sein. Bis zur Verfügbarkeit eindeutiger Belege bleibe ich dabei, dass die Aussagen zum freien Willen entfernt werden sollten.--Belsazar 23:51, 21. Okt. 2008 (CEST)
Daß ein Zufallsgenerator die Polarisationsrichtung zufällig wählen kann, wird, wie belegt, als freier Wille des Beobachters (oder des Meßapparates) gewertet. Über absichtsvolle Mißdeutungen der Formulierung des Artikels braucht hier keine philosophische Grundsatzdiskussion geführt werden. --Norbert Dragon 00:15, 22. Okt. 2008 (CEST)
  • Zufallsgeneratoren haben keinen "freien Willen". Wörtlich genommen, handelt es sich bei der Zuordnung der Eigenschaft freien Willens zu Zufallsgeneratoren um einen klassischen Kategoriefehler. Die (hier anscheinend gemeinte) metaphorische Verwendung philosophischer Begriffe birgt das offensichtliche Risiko von Missverständnissen und wirft mehr Fragen auf, als sie beantwortet.
  • Ich bin skeptisch, ob die Prämisse "freien Willens" tatsächlich eine vom lokalen Realismus unabhängige Prämisse ist. Wäre ein hypothetischer verborgener Mechanismus, der die Einstellungen des Experiments beeinflusst, nicht letztlich eine verborgene Variable, die im Bellschen Theorem bereits durch den "lokalen Realismus" operationalisiert ist?
  • Wo sind die Belege?
Zur weiteren Vorgehensweise: Ich habe den Eindruck, dass wir hier nicht recht weiterkommen. Vielleicht kann das Physik-Portal helfen -> siehe hier.--Belsazar 08:07, 22. Okt. 2008 (CEST)
in diesem zusammenhang unreflektiert von freiem willen zu sprechen ist theoriefindung und garantiert nicht der weg etabliertes wissen zu dokumentieren. wenns zu dem thema lit gibt, so ist sie sicherlich höchstdikutabel und eher was für philosophen und theologen. es wäre sehr interessant sich darüber gedanken zu machen, nicht aber hier und nicht so flach. bitte wegmachen. --Pediadeep 16:27, 22. Okt. 2008 (CEST)
Jedes anstößige frei habe ich durch zufällig ersetzt. So ist es nämlich gemeint. --Norbert Dragon 17:14, 22. Okt. 2008 (CEST)

Sorry, dass ich nicht sofort geantwortet habe, aber ich war recht beschaeftigt. Der Punkt, dass die Messrichtungen zufaellig sind (und deshalb Bells Argument in einer deterministischen Welt nicht funktioniert) ist aber schon erwaehnenswert. Zum einen, sind einige Leute schon darauf reingefallen (siehe [2] oder [3]), zum anderen begruenden andere damit die Notwendigkeit von Bell Experimenten mit Menschen anstatt Zufallsgeneratoren (und dementsprechend mit grossen Entfernungen, ob das realistisch ist, sei mal dahingestellt). Wie man das nennt, ist natuerlich sekundaer, in englischsprachigen (philosophischen oder physikalischen) Literatur ist der Begriff "free will" aber durchaus ueblich (z.B. [4] oder [5] oder fuer ganz viele Beispiele die allwissende Müllhalde: [6]). Gruss --Weissenburg 05:05, 14. Nov. 2008 (CET)

... und Einstein hat doch recht ...

Die von Einstein geforderte lokal-realistische Theorie mit verborgenen Variablen wäre nur dann widerlegt, wenn tatsächlich ein Widerspruch zwischen Bellscher Ungleichung und quantenmechanisch berechneten Mittelwerten bestünde. Dies ist, wie auch am vorliegenden Beispiel gezeigt werden kann nicht der Fall. Der vermeintliche Widerspruch ist vielmehr eine Folge von Unzulässigkeiten beim Vergleich von Mittelwerten der Bellschen Ungleichung mit solchen der Quantenmechanik.
Im Einzelnen: Die der Ungleichung zugrunde liegende Ausgangsgleichung: bezieht sich auf ein bestimmtes, verschränktes Photonenpaar mit dem Index i, an dem 3 Messungen (oder Zuweisungen von Messungen ) vorgenommen werden. Die Werte und sind daher auf der linken wie auf der rechten Seite der Ungleichung demselben, in Richtung polarisierten Photonenpaar zugeordnet. Diese triviale Feststellung darf bei der quantenmechanischen Mittelbildung nicht außer Acht gelassen werden. Der vermeintliche Widerspruch zwischen den Mittelwerten der Ungleichung

und jenen der Quantenmechanik ergibt sich daraus, dass beim Vergleich in den beiden ersten Termen die Polarisierung in Richtung festgelegt wird, im dritten Term hingegen quantenmechanische Mittelwerte von Photonen mit einer Polarisierung in Richtung bzw. gegenübergestellt werden, was einer unzulässigen Vermengung verschiedener Ensembles entspricht. Die Problematik wird leichter durchschaubar, wenn man die Ausgangsgleichung

umschreibt. Da voraussetzungsgemäß alle Werte nur die Werte +1 oder -1 annehmen können, darf für den dritten Term die Form

substituiert werden. Den Werten der linken Seite +1 bzw. -1 entsprechen genau dieselben Werte der rechten Seite. Für die Ausgangsgleichung ergibt sich damit vor der Mittelbildung

oder
,

eine Form, welche die Korrelation zwischen und aufzeigt. Bei der anschließenden Summierung verschwinden alle Glieder in welchen ist. Die Summe besteht ausschließlich aus Gliedern für welche gilt. Unter Berücksichtigung, dass sich alle Messungen auf Photonen beziehen, welche in Richtung polarisiert sind, folgt schließlich mit , und der Mittelwert

.

Setzt man andererseits in der Ausgangsgleichung

so erhält man in analoger Weise

.

Zwischen diesen beiden Ungleichungen und der Quantenphysik besteht kein Widerespruch! Der vermeintliche Widerspruch ist ausschließlich auf Verstösse gegen die Gesetze der Mengenlehre zurückzuführen.
Bravo Einstein!
H.=Ogris, h_ogris@gmx.net (Der vorstehende, falsch signierte Beitrag – siehe dazu Hilfe:Signatur – stammt von 128.131.227.225 (DiskussionBeiträge) 15:47, 4. Jan. 2009 (CET)) --128.131.227.225 10:54, 5. Jan. 2009 (CET)H. Ogris

Für den Fall, dass (zwei oder mehr) aufeinanderfolgende Messungen an einzelnen Teilchen in Betracht gezogen würden müssten zwischenzeitliche Zustandsreduktionen berücksichtigt werden. Eine konsequente Rechnung mit bedingten Wahrscheinlichkeiten im Rahmen der Quantenmechanik ergibt dann ebenfalls eine Schranke von 2 (und nicht ) und ist daher mit dem klassichen Fall identisch. Es gäbe dann, wie Du bereits erwähnt hast, keinen Widerspruch zwischen klassicher und quantenmechanischer Variante.
John Bell betont jedoch in seinem „reply to crtics“ gegenüber Peña, Cetto und Brody (Kap.8, Speakable and Unspeakable..., S.62), dass bei seiner Herleitung jeweils keine zwei Messungen an ein und demselben Teilchen vorgenommen werden dürfen. Es wird jeweils nur genau einmal gemessen.
Wenn daher im quantenmechanischen Fall keine zwischenzeitlichen Zustandsreduktionen vorgenommen werden, dann ergibt sich dort der korrekte Wert als kleinste obere Schranke, die auch erreicht werden kann.
Die Gleichheit der Indizes ist bei der Herleitung der klassichen Ungleichung von Bell auch nicht zwingend. Alternativ könnte man auch jeweils unterschiedliche Indizes (Teilchenpaare) nehmen und den gesamten algebraischen Ausdruck als eine einzige Zufallsgröße betrachten (die einer gewissen Wahrscheinlichkeitsverteilung gehorcht). Für Mittelwerte über endliche Stichprobengrößen wäre die Ungleichung dann zwar nicht beweisbar (wie Du bereits angedeutet hast). Hingegen würde die Ungleichung jedoch für unendliche Stichproben, d.h. für , mit Wahrscheinlichkeit 1 wieder zutreffen. Grüße, T.S. 17:56, 5. Jan. 2009 (CET)
Vorsicht ist geboten: Was im ersten Beitrag als "unzulässige Vermengung" abqualiziert wird, ist genau der springende Punkt am EPR-Paradox. - MfG, 87.160.106.40 16:07, 14. Jan. 2009 (CET)
Was meinst Du genau? Gruß, T.S. 23:38, 14. Jan. 2009 (CET)

Bellsche Ungleichung

Gehen wir davon aus: Die Bellschen Ungleichungen sind Aussagen (Schranken von Stückzahlen bzw. Wahrscheinlichkeiten) über Elemente eines Ensembles , welche jeweils drei Eigenschaft (A, B, C) mit ihrerseits jeweils zwei Ausformungen (+, -) besitzen. Handelt es sich um abstrakte Ensembles, sind die Elemente unveränderlich durch Definition bestimmt.

Beispiel: Ensemble: Ebene euklidische Geometrie
Eigenschaften: A, Gestalt; B, Größe; C, Orientierung
Ausformung: +, Kreis, -,Quadrat; +, groß, -, klein; linkshändig, -, rechtshändig.

Besteht das Ensemble aus Elementen der realen Welt, müssen die Eigenschaften gemessen werden. Sie treten dabei über Wechselwirkungen mit dem Anwender der Ungleichung in Verbindung. Handelt es sich um Elemente der Makrowelt, sind die unvermeidlichen Rückwirkungen dieser Messungen auf die Elemente in der Regel geringfügig und vernachlässigbar.

Beispiel: Ensemble: Bausteine
Eigenschaften: A, Gestalt; B, Größe: C, Material
Ausformung: +, Quader, -, Würfel; +, groß, -, klein; +, Beton, -, Stein.

Besteht das Ensemble jedoch aus sehr kleinen Elementen, wie Quanten, sind die Rückwirkungen nicht zu vernachlässigen. Nach dem Gesetz von Aktion und Reaktion werden Quanten verändert.

Beispiel: Ensemble: Polarisierte Photonen
Eigenschaften: Richtung A; Richtung B; Richtung C;
Ausformung: +, durchgehend; -, abgelenkt.

Die Veränderung polarisierter Photonen ist entsprechend dem Gesetz von Malus gut gesichert und erlaubt die Auswirkungen auch einer Folge von Messungen mit Filtern oder Strahlteilern auf die Wahrscheinlichkeiten an einem einlnen Photon zu berechnen. Dabei zeigt sich dass verschiedene Reihungen der Messung (z.B. C,B,A statt A,B,C) verschiedene Ergebnisse mit sich bringen. Es ist daher bei allen Vergleichen von Quantenmessungen stets die gewählte Reihung beizubehalten oder das Mittel aller möglichen Reihungen zu berücksichtigen (Faynman). Verstösse gegen diese Regel sind die Ursache unvermeidlicher Widersprüche. Verborgene Variablen müssen nicht auf fixe, unveränderliche Listen beschränkt sein. Sie können auch Anweisungen enthalten, wie sich diese im Falle einer Messung zu verändern haben. Mit dieser Art von Variablen (welche von Bell offensichtlich ausgeschlossen wurden) lassen sich viele Beobachtungen von Photonen erklären. Die entscheidende Frage ist jedoch, ob diese Erklärungen nur der Wahrscheinlichkeit genügen oder ob sie in jedem Einzelfall zwingend sind. Vertraut man den Beobachtungen an verschränkten Photonten, bei welchen Photon 1 und Photon 2 stets die gleichen Ergebnisse lieferten wenn an ihnen exakt die gleichen Messungen vorgenommen wurden, so ist dies wohl nur sehr schwer ohne verborgene Variablen zu erklären. Oder hat jemand andere Vorschläge? Zusatz: Unzulässige Gleichungen oder Variablen sind solche, welche außerhalb ihres Gültigkeitsbereches angewendet werden. Beispiel: Eine Prognoseformel für die Apfelernte ist, auch wenn sie mathematisch keine Fehler aufweist, für die Ermittlung künftiger Bananenernten unzulässsig. --91.128.116.48 16:51, 23. Jan. 2009 (CET)--91.128.116.48 16:51, 23. Jan. 2009 (CET)H. Ogris

Bellsche Ungleichung - warum sie korrekt ist, aber die Schlussfolgerungen falsch sind

Ich habe mir die Bellsche Ungleichung mal angeschaut, nachdem ich davon gelesen habe und bin zum Ergebnis gekommen, dass da irgendwas nicht stimmen kann und möchte das nachfolgend erklären:

Um das ganze einfacher und verständlicher zu machen, setze ich

a, b, bzw. c = 1, wenn das Photon durchkommt und 0, wenn das Photon absorbiert wird.

Ausserdem nehme ich an, dass die Quelle (q) bereits polarisiertes Licht aussendet (dies zum einfacheren Verständnis)

Die anfängliche Formel:

 

Ist nach wie vor korrekt, denn wenn einer der Werte a, b oder c = 0 ist, ist das Ergebnis auf der linken Seite entweder 0, 1 oder –1, wenn alle Werte 1 sind, ist das Ergebnis 1.

Wenn man nun die Summe der Mittelwerte berechnen will, muss man die Wahrscheinlichkeit berücksichtigen, mit der ein Photon durch das Filter durchkommt. Und diese ist abhängig vom Winkel zwischen der Quelle (q) und dem Ziel (a, b, c) und nicht vom Winkel zwischen Ziel 1 und Ziel 2 (wie jedenfalls in diesem Wikipedia-Artikel behauptet wird). Da beide Photonen verschränkte Zwillinge sind, mag es sein, dass auch zwischen Ziel 1 und Ziel 2 eine rechnerische Beziehung besteht, die vom Winkel zwischen Quelle und Ziel 1 bzw. 2 her berechnet werden kann. Die Formel für die Berechnung dieser Beziehung ist jedoch – so nehme ich zuerst einmal an - für das Weitere nicht relevant und ich überlasse es daher gerne anderen, diese herauszufinden.

Die Formel für die Berechnung der Produkte der Messergebnisse am Ziel a1 muss somit heissen:

 

Gemäss Messergebnis ist das Ergebnis dieser Summe bzw. die Wahrscheinlichkeit, mit der das Messergebnis positiv ist, d.h. das Photon durch den Filter kommt.

  

Dasselbe gibt dann sinngemäss auch für b1, c1, a2, b2, c2.

Wenn ich diese Summen bzw. Wahrscheinlichkeiten in die Formel der Bellschen Ungleichung einsetze, bekomme ich somit:

 

Da alle Cosinus-Werte sich zwischen 0 und 1 bewegen, ist diese Ungleichung nach wie vor korrekt

Da die Formel keine Abhängigkeiten zwischen Ziel 1 und Ziel 2 enthält, besteht somit keine Notwendigkeit, anzunehmen, dass das Ergebnis am Ziel 1 in irgendeiner Weise abhängt vom Ergebnis am Ziel 2.

Ausserdem gibt es somit keinen Grund, anzunehmen, dass die quantenmechanischen Wahrscheinlichkeiten nicht von der Unkenntnis verborgener Parameter herrühren

(Wenn man jetzt davon ausgeht, dass die Quelle kein polarisiertes Licht aussendet, könnte man sich ja auch eine Reihe von Experimenten vorstellen, bei denen man die Quelle in jeder beliebigen Richtung polarisiert; das Ergäbnis wäre natürlich dasselbe).

(nicht signierter Beitrag von 87.102.168.45 (Diskussion | Beiträge) 00:46, 2. Feb. 2009 (CET))

Pro: Überarbeitung "Herleitung".

Ich hatte gerade beim Lesen des Artikel den selben Gedanken. Zusätzlich würde ich eine
Skizze vorschlagen. Das wäre eine Hilfe um es visuell zu verstehen.
VG,
forky
(nicht signierter Beitrag von forky/IP (Diskussion | Beiträge) 11:35, 23. Okt., 2009)

Naja

3 Ziele führt Bell nur ein, weil in keiner Weise festgestellt werden kann wie die "Quelle" polarisiert ist noch die Photonen "an der Quelle" polarisiert werden können. Man kann die Polarisierung an der Quelle nicht kontrollieren. Indem du nun also den gesuchten Wert von Anfang an einführst änderst du das gesamte Problem. Es ist schön dass du mit Bell's Ungleichung so hübsche Dinge tun kannst aber um Bell's Ungleichung zu widerlegen musst du Bell's Ungleichng widerlegen. (nicht signierter Beitrag von 128.179.130.186 (Diskussion) 21:44, 21. Nov. 2012 (CET))

Lokalität ungenau definiert

In der Definition von Lokalität sehe ich eine Ungenauigkeit und einen klaren Fehler. Erstens ist es nicht einfach die Wahl, die sich auswirkt oder nicht, sondern die Messung. Es soll offenbar betont werden, dass vom Akt des Messens (der natürlich eine vorher gewählte Observable anspricht) und nicht vom Messergebnis die Rede ist. Die Wahl (Entscheidung), was gemessen werden soll, fällt aber in der Regel vor der Messung. Darum finde ich die Formulierung undeutlich.

Zweitens ist das Wort ‚augenblicklich‘ ein echter Fehler. Ein Zeitbezug ohne Angabe eines Inertialsystems bedeutet ja, dass eine absolute Zeit vorausgesetzt wird. Ob die physikalischen Gesetze unserer Welt lokal oder nichtlokal sind, muss aber doch (zumindest auch) unter Annahme der speziellen Relativitätstheorie diskutiert werden; Newton ist nicht mehr aktuell.

Das kann ich beides, wie ich denke, im Sinne des Autors korrigieren.

Da ist aber noch eine inhaltliche Frage, der ich nicht weiter nachgehe. Vielleicht weiß jemand anderes was dazu: Wenn eine Messung (hier und jetzt) mir Information liefern könnte über ein entferntes Objekt, ohne auf dieses einen ‚Einfluss‘ auszuüben, wäre das nicht auch ein nichtlokales Phänomen? Wird aber von dieser Definition nicht abgedeckt! Außerdem ist das Wort ‚Einfluss‘ undefiniert.-- Binse (Diskussion) 23:32, 21. Aug. 2013 (CEST)

P.S. Das Wort ‚weit‘ habe ich aus ‚räumlich weit entfernt‘ gestrichen. Da es keine Grenze zwischen ‚weit entfernt‘ und ‚nicht so weit entfernt‘ gibt, darf das in der Definition nicht stehen.-- Binse (Diskussion) 00:04, 22. Aug. 2013 (CEST)

Herleitung falsch

Die Aussage "Da bei gleicher Richtung beider Filter das eine Photon genau dann absorbiert wird, wenn auch das andere Photon absorbiert wird, gilt in allen Fällen" ist falsch. Das gilt im statistischen Mittel aber nicht für jede einzelne Messung. (nicht signierter Beitrag von 93.207.192.131 (Diskussion) 01:40, 24. Nov. 2013 (CET))

Doch, es gilt für jede einzelne Messung. Und das ist genau der Punkt, auf den es bei der Verschränkung ankommt.---<)kmk(>- (Diskussion) 09:15, 24. Nov. 2013 (CET)
Die Argumentation der IP ist schon logisch und statistisch falsch, dazu braucht es noch nicht mal die konkrete Physik.
  • „Genau dann, wenn“ heißt 1. „Nur wenn“ und 2. „Immer wenn“. „Immer wenn“ heißt aber schon logisch „jedes einzelne Mal“.
  • Auch statistisch verlangt ein Durchschnitt eines Merkmals von 100 Prozent, dass jedes einzelne in den Durchschnitt einfließende Element individuell ebenfalls 100 Prozent dieses Merkmals aufweist, denn auch nur ein einziges Element von unter 100 Prozent verhindert, dass der Durchschnitt 100 Prozent beträgt. Wenn etwas „im statistischen Mittel immer“ so ist, dann ist es notwendigerweise jedes einzelne Mal so.
Die Aussage, „‚genau dann, wenn‘ gilt nur im statistischen Mittel, aber nicht in jedem Einzelfall“ ist daher keine sinnvolle Aussage. Troubled @sset  Work  Talk  Mail   10:45, 24. Nov. 2013 (CET)

Die IP hat teilweise recht, das gilt für , wenn das eine photon spin up hat in richtung a, hat das andere spin down in dieselbe richtung, geht also auch durch den Filter längs a. Misst man mit gedrehten filtern b erhält man immer nur Wahrscheinlichkeitsaussagen aus der QM. Das ist aber in erster Linie ein Formulierungsproblem im Text.--Claude J (Diskussion) 10:50, 24. Nov. 2013 (CET)

Herleitung falsch

Die Aussage "Da bei gleicher Richtung beider Filter das eine Photon genau dann absorbiert wird, wenn auch das andere Photon absorbiert wird, gilt in allen Fällen" ist falsch. Das gilt im statistischen Mittel aber nicht für jede einzelne Messung. (nicht signierter Beitrag von 93.207.192.131 (Diskussion) 01:40, 24. Nov. 2013 (CET))

Doch, es gilt für jede einzelne Messung. Und das ist genau der Punkt, auf den es bei der Verschränkung ankommt.---<)kmk(>- (Diskussion) 09:15, 24. Nov. 2013 (CET)
Das muss bezweifelt werden. Denn wenn bei jeder einzelnen Messung bei beide Photonen das gleiche Verhalten zeigen würden, hätte man eine Möglichkeit überlichtschneller Kommunikation: Man könnte durch Mehrfachmessungen des Polarisationswinkels bei Bob auf die Stellung des Polarisators bei Alice schließen, und das über beliebige Entfernungen. Diese einfache Möglichkeit wäre eine technische Sensation und sicher schon realisiert, wenn es ginge. Aus diesem Grunde ist die im Artikel widergegebene Herleitung zu hinterfragen. 89.245.108.189 01:45, 13. Mai 2015 (CEST)
Oh, das ist ein wichtiger Punkt, der hier ganz genau diskutiert werden muss! M.E. ist der Mangel der Herleitung, dass man gar nicht konkretisiert, was für ein Zustand hier gemeint ist. Es gibt ja nicht einfach genau einen verschränkten Zustand. Der Punkt ist natürlich die perfekte Korrelation der beiden Messungen in einem maximal verschränkten Zustand. Dadurch liegt bei jeder Einzelmessung das Messergebnis von Bob bei Kenntnis des Messergebnisses von Alice schon vor, wenn beide Polarisatoren dieselbe Einstellung haben. Das allein ist ja nichts Verwerfliches, sondern ein ganz normaler statistischer Vorgang. Wenn man Schuhpaare einzeln verpackt, Alice ein Paket öffnet und einen rechten Schuh sieht, weiß sie auch, dass Bob einen linken Schuh sehen wird. Das Besondere am maximal verschränkten Zustand hier ist, dass die maximale Korrelation unabhängig davon auftritt, welchen Winkel der Polarisator hat. Aber auch damit lässt sich keine überlichtschnelle Kommunikation bewerkstelligen. Alice weiß zwar in jedem Einzelexperiment, dass Bob genau dasselbe misst wie sie, aber nur, wenn sie die Stellung von Bobs Polarisator bereits kennt. Die perfekte Korrelation zwischen zwei Messreihen von Alice und Bob kann nur durch Austausch der Messdaten (klassische Kommunikation) experimentell überprüft werden, so dass sie nicht auf die Stellung von Bobs Polarisator durch Einzelmessung schließen kann. --CWitte (Diskussion) 07:39, 13. Mai 2015 (CEST)'
Wie kann man sich eigentlich den physikalischen Grund vorstellen, der die vollständige Korrelation bei beliebigen Winkeln gewährleistet? - Angenommen, Bob ist einen Meter (oder auch eine Lichtminute) weiter von der Photonenquelle entfernt als Alice. Kommt es dann immer noch zur vollständigen Korrelation beim Verhalten der Photonen? - Auf welche Weise bestimmt die Messung bei Alice, wie sich das Photon bei Bob verhalten wird? (Im Falle der Lichtminute eine Minute später.) Ist es die "spukhafte Fernwirkung" oder doch ein physikalisches Phänomen wie z.B. die Polarisation der Photonen, die von Alice festgestellt wird? Wird das Photon auf dem Weg zu Bob etwa im Fluge polarisiert, noch bevor es schließlich bei Bob ankommt? 89.245.76.99 00:32, 14. Mai 2015 (CEST)
Zitat aus dem QuantumLab der Uni Erlangen (die letzten beiden Absätze auf der Seite):
Gleiche Winkel der λ/2 Platten:
Wenn das Photon z. B. bei Alice bei einem Winkel der λ/2 Platte von 45° transmittiert wird, so nimmt instantan das Photon bei Bob auch den Winkel von 45° an. Wenn der Winkel der λ/2 Platte bei Bob ebenfalls auf 45° eingestellt ist, so wird das Photon mit 100% Wahrscheinlichkeit transmittiert.
Wenn das Photon z. B. bei Alice bei einem Winkel der Wellenplatte von 45° reflektiert wird, so nimmt instantan das Photon bei Bob den Winkel von 45° - 90° = - 45° an. Wenn der Winkel der Wellenplatte bei Bob wieder auf 45° eingestellt ist, so wird das Photon mit 0% Wahrscheinlichkeit transmittiert. Beide Photonen werden reflektiert.
Sobald die Winkel der beiden λ/2 Platten übereinstimmen verhalten sich die Photonen exakt gleich. Die Entscheidung des "ersten" Photons bei Alice für transmittiert oder reflektiert ist ein reiner Zufallsprozess.
Verschiedene Winkel der λ/2 Platten:
Bei verschiedenen Einstellungen der Winkel an den λ/2 Platten wird zunächst die Polarisation des einen Photons durch eine Messung festgelegt. Das andere Photon nimmt instantan die entsprechende Polarisation an. Durch die Messung eines einzelnen Photons des Paares wird die Verschränkung untereinander zerstört. Das noch nicht gemessene Photon hat nun eine fest definierte Polarisationsrichtung und verhält sich dementsprechend wie ein einzelnes Photon mit entsprechender Anfangspolarisation an der Kombination λ/2 Platte und Strahlteiler. (nicht signierter Beitrag von 89.245.114.203 (Diskussion) 15. Mai 2015, 01:36)

Traurig, traurig, dass so was mit dem Siegel einer Universität daherkommt. Die drei Sätze, die im vorangehenden Abschnitt fett gedruckt sind, also wohl Zitate darstellen, sind falsch. Rufen wir uns ins Gedächtnis, dass verlässliche Erkenntnis nur durch Messungen gewonnen wird. Was am hier diskutierten Experiment physikalische Relevanz hat, sind einzig die Messwerte von Alice und Bob und dabei vor allem die Korrelation zwischen Alices und Bobs Messungen, der (von der Stellung der Polfilter zueinander abhängige) Prozentsatz an Übereinstimmungen von transmittiert/reflektiert. Was zwischen den Messungen geschieht (etwa, ob und wann die Polarisation eines Photons verändert wird), ist dagegen Spekulation, in etwas gröberer Sprache Phantasie. Was die quantumlab-Leute hier behaupten, lässt sich auch mit einem erweiterten Experiment nicht prüfen. Das ist gar keine Physik.

Auch wenn's ein paar Zeilen mehr werden, sollte ich das wohl deutlich machen. Wenn man an das instantane Polarisieren von Bobs Photon glaubt, müsste sich der Punkt der Flugbahn, an dem das passiert, ja feststellen lassen. Wir wiederholen das Experiment also vielfach, rücken dabei Bobs Filter dem Flug seines Photons entgegen, und schauen jedes mal nach, ob es schon die Polarisation von Alices Photon zeigt. Jedoch: April, April! Es gibt gar keinen Punkt, an dem sich etwas ändert: Die gleiche Orientierung der Filter bedingt, wie wir doch wissen, stets die Gleichheit der Messwerte!

Die instantane Einstellung des Photons auf eine bestimmte Polarisationsebene ist also Phantasie, oder höflich gesagt, Interprätation von quantumlab. Und dazu eine schlechte Interprätation, denn ‚Instantan‘ heisst doch sofort, gleichzeitig. Bei voneinander entfernten Ereignissen gibt es aber, wie jeder Physiker weiss, keine absolute Gleichzeitigkeit, sondern Gleichzeitigkeit hängt vom Bewegungszustand des Beobachters (seinem Inertialsystem) ab. Für verschiedene Beobachter müsste das Photon seine Polarisation an verschiedenen Punkten seiner Flugbahn ändern. Diese Interprätation, Spekulation, Fantasie, taugt offenbar nicht viel. Physik ist das jedenfalls nicht.- Binse (Diskussion) 01:33, 22. Jul. 2015 (CEST)

Diese Interpretation taugt ziemlich viel. Sie spricht der Wellenfunktion eine objektive Realität ab, aber sie ist wunderbar im Einklang mit Experimenten. Ja, verschiedene Systeme beschreiben das Experiment verschieden, aber alle kommen zum gleichen Schluss, was Messwerte angeht. --mfb (Diskussion) 13:04, 22. Jul. 2015 (CEST)
Dass der Kalkül richtige Vorhersagen für die Messwerte macht, bezweifelt ja niemand. Es geht mir um die Interprätation des Geschehens zwischen den Messungen. Die ist und bleibt Phantasie und ist ja für die Vorhersage des Messwertes (Betragsquadrat der Wellenfunktion am angenommenen Raum/Zeit-Punkt gibt die Wahrscheinlichkeit) überflüssig. Das sie als Denkmodell taugt, kann ich nicht sehen. Du sagst ja selbst, dass keine Realität dahinter steht. In einfachen Fällen mag sie unschädlich sein und sogar nützlich im Sinne einer Eselsbrücke; aber bei der Diskussion eines verschränkten Teilchenpaars kannst Du aus den Diskussionen dieser Seite sehen, wie sinnlos es ist, Behauptungen ohne Realitätswert aufzustellen, die sich hoffnungslos widersprechen. Das kann nur Verwirrung stiften. Bei einem Versuchsaufbau, der Messungen an Raum/Zeit-Punkten in raumartigem Verhältnis verwendet, verbietet sich z.B. das Wort instantan ganz von selbst und zwar absolut, weil es in jedem Fall zu unsinnigen Ergebnissen führt.- Binse (Diskussion) 14:06, 27. Jul. 2015 (CEST) Schlampige Formulierung korrigiert, Binse (Diskussion) 15:15, 30. Jul. 2015 (CEST)
Nach dieser Erklärung würde das Photon tatsächlich auf dem Weg zu Bob im Flug polarisiert. Die instantane Polarisierung entspricht der Stellung des Polarisators bei Alice, wenn dort das andere Photon polarisiert wird. Wenn nun Bob den gleichen Winkel wie Alice bei seinem Polarisator eingestellt hat, verhält sich sein Photon identisch zu dem anderen Photon bei Alice, auch wenn es zu einem späteren Zeitpunkt bei Bob ankommt. --89.245.114.203 01:36, 15. Mai 2015 (CEST)
Überlichtschnelle Kommunikation möglich?!
Wenn wir also davon ausgehen können, dass bei der Messung durch Alice auch die Polarisation des in Richtung Bob fliegenden Photons festgelegt wird, haben wir den entscheidenden Mechanismus für die überlichtschnelle Kommunikation: Alice und Bob vereinbaren, dass Alice an ihrem Polarisator nur die Winkel 0° oder 45° einstellt. Wenn Alice ein Bit mit Wert 0 sendet, stellt sie 0° ein. Entsprechend würde sie ein Bit mit dem Wert 1 durch die Winkeleinstellung 45° senden.
Sobald nun ein Photon durch Alice' Polarisator fliegt, wird die Polarisation beider Photonen festgelegt. Auch das Photon Richtung Bob wird polarisiert. Bei dem Bitwert 0 hätte Bobs Photon horizontale Polarisierung oder auch vertikale, wenn Alice' Photon im Polarisator absorbiert wurde. Bei dem Bitwert 1 wäre Bobs Photon diagonal polarisiert (Winkel 45° oder 135° bzw -45°). Nun muss Bob nur noch feststellen, welche der beiden Möglichkeiten (horizontal/vertikal oder diagonal polarisiert) bei dem ankommenden Photon vorliegt. --89.245.77.132 00:24, 16. Mai 2015 (CEST)
Egal was sie bei den Polarisatoren einstellen, sie werden immer zu 50% "fliegt durch" und zu 50% "fliegt nicht durch" messen. Die Verschränkung ergibt nur eine Korrelation der Messergebnisse, die lässt sich aber nicht ausnutzen da man dieses Messergebnis nicht festlegen kann. Bob kann nicht ermitteln, ob eine diagonale oder [vertikale oder horizontale] Polarisation vorliegt. --mfb (Diskussion) 00:29, 16. Mai 2015 (CEST)
Die ganze Diskussion beruht m.E. auf der vielfach (auch im vorliegenden Artikel!) anzutreffenden falschen Behauptung, die Messung an einem Teilchen bewirke irgend etwas an dem anderen. Die Nichtlokalität der Naturgesetze stellt unserem Denken da eine Falle, in die man tatsächlich leicht hinein tappt. Ich versuche das zu erklären.
Die Theorie verlangt, und, soviel ich weiss, bestätigt das Experiment (wie immer im Rahmen der Messgenauigkeit), dass bei entsprechender Aufstellung der Polarisationsfilter Alice und Bob jeweils die gleiche Beobachtung machen: Bei beiden passiert das Photon das Filter oder bei beiden wird es absorbiert. Natürlich liegt es jetzt nahe, zu denken, das Eine sei eine Wirkung des Anderen. Aber, da Alice und Bob für die Naturgesetze wohl gleichberechtigt sind: Wessen Messung soll denn da die andere beeinflusst haben? Man kann ja mal vermuten, wer zuerst misst, bestimmt das Ergebnis des Anderen mit. Das passt aber leider nicht zur speziellen Relativitätstheorie, die bekanntlich durch eine überwältigende Masse von Beobachtungen bestätigt ist. Die wirklich interessanten Experimente sind doch die, bei denen Alice und Bob sich ihre Ergebnisse nur mit Überlichtgeschwindigkeit vor der Messung mitteilen könnten. Wenn beispielsweise die Filter je genau einen km in entgegengesetzten Richtungen stehen. Die beiden Labors denken wir natürlich feststehend. Dann ist die Zeitdifferenz zwischen beiden Messungen nur so gross, wie das Licht braucht, die Differenz der beiden Strecken, also vielleicht ein paar cm, zu durchlaufen. Für derartige zueinander raumartige Messpunkte gibt es aber gar keine vorher/nachher Abfolge. Wer von beiden eher misst, hängt vom Bewegungszustand des Beobachters ab. Im Flugzeug in einer Richtung die Messstrecke abfliegend sieht man Alice als erste messen, wer entgegengesetzt fliegt, stellt fest, dass Bob eher misst. Beharrt man darauf, dass die erste Messung die andere beeinflusst, dann hängt die Richtung des Einflusses vom Beobachter ab. Eine Physik, in der das Geschehen vom Beobachter abhängt, dürfte aber noch weit schwieriger sein, als eine, die ‚nur‘ nichtlokal ist.
Ich nenne einen zweiten Grund, warum keine der Messungen die andere beeinflusst. Beide Experimentatoren beobachten, dass das jeweilige Photon mit 50% Wahrscheinlichkeit passiert oder verschluckt wird. Mit anderen Worten, sie beobachten eine Zufallsfolge von ‚passiert‘ und ‚passiert nicht‘. Wenn Sie oder Er aber mal das Messen einstellen, etwa das Filter abdecken, beobachtet Er bzw. Sie weiterhin eine Zufallsfolge, wie beschrieben, und hat keine Möglichkeit festzustellen, ob bei Ihr bzw. Ihm überhaupt eine Messung stattfindet. Wenn man darauf beharrt, dass z.B. ihre Messung seine beeinflusst, muss man also akzeptieren, dass dieser Einfluss sich in keiner Weise nachweisen (beobachten) lässt. Effekte, die sich prinzipiell nicht beobachten lassen, sind aber keine Physik. Mit genau diesem Argument hat Einstein den Lichtäther aus der Physik verbannt.
Fazit: Messungen an verschränkten Teilchen ergeben korrelierte Werte, bis hin zur vollständigen Übereinstimmung. Dabei besteht aber zwischen den Messwerten kein Ursache/Wirkung-Verhältnis, und die zeitliche Ordnung der Messungen ist unerheblich. Damit wird auch verständlich, dass die Naturgesetze nichtlokal sind, ohne dass sich daraus überlichtschnelle Wirkungen ergeben.- Binse (Diskussion) 03:13, 20. Jul. 2015 (CEST)

Enzyklopädische Beschreibung

da steht: "Die Bellsche Ungleichung ist eine Schranke an Mittelwerten von Messwerten, die 1964 von John Bell angegeben wurde."

http://www.scholarpedia.org/article/Bell%27s_theorem "Bell's theorem asserts that if certain predictions of quantum theory are correct then our world is non-local. "Non-local" here means that there exist interactions between events that are too far apart in space and too close together in time for the events to be connected even by signals moving at the speed of light."

habt Ihr noch alle, unbedarfte Grenzgänger zu quälen mit "Schranke an Mittelwerten von Messwerten" ? Welcher Mensa-Idiot kommt auf solch eine karikaturistische Idee des Unverständnisses?

das ist eine Enzyklopädie und kein Abstellgleis für Nichtartikulationen!--91.34.252.202 02:46, 8. Feb. 2014 (CET)

Guter Hinweis. Ich habe das mal entsprechend angepasst. --B wik (Diskussion) 13:24, 24. Okt. 2015 (CEST)

Definition von ,lokal' problematisch

Wir lesen da:

Eine physikalische Theorie ist lokal, wenn sich bei zwei räumlich getrennten Teilchen die Wahl dessen, was beim einen Teilchen gemessen wird, bei der Messung nicht unmittelbar auf das andere Teilchen auswirkt.

Obwohl ich leider nichts besseres anbieten kann, mit anderen Worten kein Zitat anbieten kann, das mir besser gefällt, möchte ich darauf hinweisen, dass es nach dieser Formulierung in einer nichtlokalen Welt unmittelbare Fernwirkungen gäbe, was doch nach weitgehend einmütigem Urteil der Fachleute für die quantenmechanische Welt nicht zutrifft.

Ich habe oben in der Diskussion schon darauf hingewiesen, dass die Korrelation der Messwerte an zwei entfernten Teilchen, die es theoretisch wie experimentell zweifellos gibt, nicht als Einfluss der einen Messung auf die andere interpretiert werden kann. Wenn Alice zeitweise das Messen einstellt, hat das nicht die geringste Auswirkung auf Bobs Messwerte. Erst recht hat die Wahl dessen, was Alice misst oder nicht misst, keinerlei Einfluss. Entgegen der Intention der Definition wäre die Quantentheorie also lokal. Dass die Gleichberechtigung von Bob und Alice, sowie auch die Unbestimmtheit der zeitlichen Abfolge der beiden Messungen hier ignoriert werden, macht die Definition noch schlechter.

Wenn man schon lokal/nichtlokal an Messungen mit Teilchenpaaren festmachen will, was sicher nur einen Spezialfall darstellt, dann ist festzuhalten, dass die beobachtete Nichtlokalität im Vorliegen von Korrelationen zwischen Messwerten an Teilchenpaaren besteht, die an Raum/Zeitpunkten in raumartiger Relation vorgenommen wurden, und dies ohne dass dabei ein gegenseitiger Einfluss feststellbar ist. Vielleicht findet jemand ein passendes Zitat.

So oder so: Eine zufriedenstellende Definition von ‚nichtlokal‘ zu geben, ist eine echte Herausforderung! - Binse (Diskussion) 02:40, 20. Jan. 2016 (CET)

Doch, nach der üblichen Interpretation der Quantenmechanik gibt es Fernwirkung (Kollaps der Wellenfunktion), nur keine instantane Übertragung von Signalen (Information), siehe auch Lokalität (Physik). Wenn A in dem Zerfalls-Paar Spin up misst kann B wegen Drehimpulserhaltung nur Spin down messen, nur weiss er von der Messung von A nur falls er ein Signal von diesem erhält. Er wird ansonsten nichts außergewöhnliches bemerken, da die Theorie (QM) nur statstische Vorhersagen macht (viele Messungen, Wahrscheinlichkeit Spin up/down eines teilchens aus dem zerfallspaar fifty fifty)--Claude J (Diskussion) 08:09, 20. Jan. 2016 (CET)
Nun ja, Claude J! Der instantane Kollaps der Wellenfunktion hat doch mit der Wirklichkeit wenig zu tun. Unbestritten ist: Man kann ihn nicht beobachten. Und was man nicht beobachten kann, hat in der Physik als Wissenschaft eigentlich nichts zu suchen (siehe Einstein und Lichtäther). Etwas, das prinzipiell nicht beobachtet werden kann, als Wirkung (in unserem Fall Fernwirkung) zu bezeichnen, ist zumindest sehr angreifbar. Ich rede hier nicht von der Korrelation der spin up/spin down Messungen, die ja beobachtbar ist. Ich rede vom Einfluss. Vom angeblichen Bewirken. Wenn A mal nicht misst, also keinen Kollaps auslöst, ändert sich bei B nicht das Geringste. Also bewirkt der Kollaps doch nichts. Man kann den instantanen Kollaps wohl am ehesten als eine zweckmäßige Sprachregelung sehen. Und die scheint mir gerade bei der Diskussion von Verschränkungsphänomenen arg an ihre Grenzen zu stoßen. Bei Lokalität (Physik) besteht natürlich ebenso Diskussionsbedarf. Zum instantanen Kollaps finde ich dort übrigens eine deutliche Distanz. - Binse (Diskussion) 16:00, 20. Jan. 2016 (CET)
Ich bezog mich auf deinen Satz, instantane Fernwirkung würde es nach einmütiger Meinung der Fachleute nicht geben. Dem ist nicht so, siehe erst jüngst Zeilingers Experiment, und Anton Zeilinger gilt als Koryphäe auf dem Gebiet (der hat sogar ein Buch geschrieben mit dem Titel: Spukhafte Fernwirkung, die Schönheit der Quantenphysik)--Claude J (Diskussion) 17:19, 20. Jan. 2016 (CET)
Man kann den Kollaps nicht beobachten, aber egal welche Interpretation man wählt: ohne etwas, das man nicht direkt beobachten kann, kommt man nicht aus (man kann aber Dinge wählen, die wesentlich naheliegender sind als magische Kollapse). Das ist gerade eine der wesentlichen Punkte der Quantenmechanik und dieser Ungleichung. --mfb (Diskussion) 17:46, 20. Jan. 2016 (CET)

Eine Frage zum Satz "„Klassische“ Theorien wie die newtonsche Mechanik ... besitzen beide dieser Eigenschaften."


Ist die Gravitationstheorie Newton's wirklich "lokal"? Ich dachte, dass sie ein typisches Beispiel einer "Fernwirkungstheorie" sei?
Im Artikel Lokalität (Physik) steht:
"Typisches Beispiel ist das klassisch-newtonsche Konzept der Gravitation, die beliebig fern und instantan wirkt."

--188.22.55.141 18:41, 24. Jan. 2016 (CET)

Newtonsche Mechanik ist wohl ohne Gravitation gemeint, aber das ist irreführend. Man könnte es durch die Allgemeine Relativitätstheorie ersetzen, aber ob die so "klassisch" ist? --mfb (Diskussion) 18:56, 24. Jan. 2016 (CET)

Identische Vorhersagen von Quantenmechanik und realistisch-lokalen Annahmen möglich

Ich glaube, eine Formel der Quantenmechanik für die Wahrscheinlichkeit, den Spin des 2.Teilchens von 2 Teilchen mit verschränktem Spin in einer bestimmten Orientierung zu finden, mit Annahmen der Realität und Lokalität über die Natur des Spins ableiten zu können. Wo bleibt dann noch die Berechtigung für die Bellsche Ungleichung?

Genaue Definition der Wahrscheinlichkeit P: Teilchen 2 mit + Orientierung gemessen bei Vorzugsrichtung θ für alle zugehörigen Teilchen 1 mit - Orientierung für Vorzugsrichtung 0 (keine Verdrehung)

Die Formel lautet: P = 1/2 * (1 + cos θ) und wurde entnommen: Sakurai, J.J.; Modern Quantum Mechanics. Hier stellt θ den Winkel zw. der Vorzugsrichtung bei der Messung des Spins von Teilchen 1 und der des Teichens 2 dar. Die gleiche Formel leite ich ab mittels Annahmen der Realität und Lokalität.

--2003:C3:6707:6900:5014:1583:9FFE:8D72 13:34, 27. Dez. 2017 (CET)

WP:Theoriefindung. Publiziere es, wenn du denkst etwas neues gefunden zu haben (Tipp: Nein, hast du nicht). Hier ist nicht der richtige Ort dafür. --mfb (Diskussion) 21:29, 27. Dez. 2017 (CET)

Okay, war mir so nicht klar. Ich dachte, Diskussion bezieht sich auf das Thema allgemein, nicht auschließlich auf den Artikel.

Neue Einleitung?

Die Einleitung des Artikels finde ich in so vielen Punkten, sagen wir ‚unschön‘, dass ich mal eine Neufassung vorschlage. Wo stehen in der Ungleichung Korrelationen? Mit dem Wort klassisch wird ganz unbegründet die Relativitätstheorie ausgeblendet, das Wort realistisch, das unbedingt vorkommen sollte, fehlt. Die Verletzung der Ungleichung ist zwar auch eine von vielen Bestätigungen der Quantentheorie, aber vor allem eine Widerlegung des lokal-realistischen Standpunkts, was ich wichtiger finde. Vor allem aber sollte deutlich mehr Inhalt für die Leser da sein, die sich den ausführlichen, technischen Teil nicht zumuten wollen. Mein Vorschlag. Ich bitte, allgemeine Zustimmung oder Ablehnung etc. auf der hiesigen Diskussionsseite auszudrücken, Kommentare, Korrekturen Verbesserungsvorschläge etc. auf der dortigen Diskussionsseite, und meinen Text selbst erst mal nicht zu ändern. Falls ich keine deutliche Ablehnung spüre, denke ich in etwa einer Woche ‚zur Tat zu schreiten‘. Auch der Artikel selbst erlaubt erhebliche Verbesserungen. Darauf möchte ich zurückkommen.-- Binse (Diskussion) 14:42, 11. Apr. 2018 (CEST)

Habe dort kommentiert (Benutzer Diskussion:Binse/Bells Ungleichung). --Zulu55 (Diskussion) Unwissen 15:13, 11. Apr. 2018 (CEST)
Korrelationen oder genauer die zugehörigen Korrelationskoeffizienten sind das was in der Bell-Ungleichung zwischen den eckigen Klammern steht, siehe auch zum Beispiel hier oder en:Quantum correlation). Wie du siehst ist das eine gebräuchliche Bezeichnung, wie man auch durch googeln leicht findet. Die Korrelationen können klassisch (verborgene Variable) oder quantenmechanisch sein. Für quantenmechanische Systeme entspricht das der Verschränkung. Ich finde deinen Text keine Verbesserung, insbesondere holt er zu weit aus und ich fand sie beim Durchlesen eher verwirrend (Zitat "bellsche Ungleichung, der die Häufigkeiten der betroffenen verborgenen Eigenschaften genügen müssten"). Die Definition in der Einleitung sollte direkt sein, auch wenn nicht alles in der Zusammenfassung klargestellt werden kann. Das überfordert die Zusammenfassung.--Claude J (Diskussion) 16:45, 11. Apr. 2018 (CEST)
Hallo Claude J! Mit den Korrelationen hast Du schon recht. Was da als Skalarprodukt steht, kann man durchaus als Korrelationen lesen. Hatte ich übersehen. Bei meiner sonstigen Kritik bleibe ich aber erst mal. Die Widerlegung der Annahme von verborgenen Eigenschaften und überhaupt des lokal-realistischen Standpunkts ist viel wichtiger als eine weitere Bestätigung der schon wohletablierten Quantentheorie. Bei dieser geht es ja um einen ganz speziellen Punkt: Ist der Zufall fundamentel? Aber vor allem sollte mehr Inhalt in die Einleitung. Was es mit den verborgenen Variablen überhaupt auf sich hat, dass die ganze Diskussion schon 35 durch EPR losgetreten, erst 64 durch Bell theoretisch und gar erst 82 durch Aspect und andere experimentell entschieden wurde, findet der Leser nicht, wenn er sich den technischen Teil nicht zumuten will. An der Verständlichkeit des Entwurfs lässt sich arbeiten. Bin jetzt leider ein paar Tage offline. Bis bald! Gruß Binse (Diskussion) 12:16, 12. Apr. 2018 (CEST)
Vor allem sollte da glasklar stehen, dass die Ungleichung für klassische Systeme erfüllt ist und in der QM verletzt werden kann, das war früher nicht so deutlich dargestellt. Die Bell-Ungleichung wird im Übrigen auch manchmal kurz so zusammengefasst, dass sie eine experimentell überprüfbare effektive (quantitative) Version des EPR-Paradoxons ist.--Claude J (Diskussion) 12:54, 12. Apr. 2018 (CEST)
Hallo Claude J und Zulu55! Bin wieder da. Schaut bitte noch mal rein. Ich habe versucht, Eure Anregungen zu berücksichtigen und meinen Entwurf dabei gestrafft. Ihr werdet ihn jetzt hoffentlich klarer finden. Nochmal kurz zu: Bestätigung der Quantentheorie. Die sehe ich hier schon deshalb nicht, weil Kalkül und Vorhersagen der Q. ja nicht davon abhängen, ob der Zufall echt ist oder durch verborgene Variablen vorgetäuscht wird. Wäre es anders, so hätten wir in der Bestätigung der Quantentheorie, die die verborgenen Variablen widerlegt, sogar einen Zirkelschluss (der dann mit der experimentellen Bestätigung allerdings gegenstandslos wird). Übrigens ist EPR für mich kein Paradoxon. Das Wort hat sich nur so eingebürgert.-- Binse (Diskussion) 00:42, 18. Apr. 2018 (CEST)
Bei den verborgenen Variablen muss natürlich noch die Eigenschaft der Lokalität ergänzt werden.--Claude J (Diskussion) 07:36, 18. Apr. 2018 (CEST)
Muss das wirklich sein?. Der Artikel über verborgene Variable macht zwar die Unterscheidung, sagt aber nichts Brauchbares über die nichtlokalen, außer dass sie in der Bohmschen Mechanik vorkommen. Ich sehe auch garnicht, wie man einer Eigenschaft eines Teilchens Lokalität zuschreiben oder absprechen kann. Ich könnte mir eine Fußnote vorstellen: „Genauer gesagt handelt es sich hier um lokale verborgene Variablen“. Für die enzyklopädische Vollständigkeit würde das jedenfalls genügen. Aber lieber würde ich das, wenn schon überhaupt, auf die Durchführung verschieben, wo ohnehin erklärt werden muss, wie die individuellen Eigenschaften die Lokalität zu retten versuchen. Dass sie verborgen genannt werden, ist ein Seitenaspekt, der mit dieser Funktion nichts zu tun hat, ein Link zur Erweiterung des Themas, nicht viel mehr als „siehe auch“. Die Einleitung soll sich doch auf das Wesentliche beschränken. Nebenbei betont das Wort verborgen auch ohne Erklärung Bells Leistung.- Was war Deine Idee?-- Binse (Diskussion) 01:41, 19. Apr. 2018 (CEST)
Verborgene Variable stand schon länger in der Einleitung, das ist aber auch der bekannteste Name für eine "klassische" (deterministische) Interpretation der QM (verwendet in dem Buch von von Neumann 1932 in dem er meinte diese mathematisch widerlegt zu haben). Einstein-Rosen-Podolsky (deren Analyse hier verwenden das nicht, möglicherweise weil damals allgemein galt das von Neumann diese Möglichkeit widerlegt habe (was spätestens seit Bell 1966 ja nicht mehr gilt). Ob das da oder unten erwähnt wird ist mir egal.--Claude J (Diskussion) 07:35, 19. Apr. 2018 (CEST)
"lokal" ist hier absolut notwendig. Lokal sind aber nicht die Variablen, sondern die Physik insgesamt (insbesondere die Frage, was diese Variablen ändern kann). --mfb (Diskussion) 07:50, 19. Apr. 2018 (CEST)
Bell bezieht sich in seinem Aufsatz von 1964 in der Einleitung selbst auf Theorien verborgener Variabler. Er verweist zuerst auf die vorgebliche Widerlegung durch von Neumann, wobei er auf seinen erst später 1966 erschienenen Aufsatz verweist, der in von Neumanns Beweis einen elementaren Fehler nachweist, und stellt dann heraus, dass er in diesem Aufsatz zeigen will, dass die eigentliche Grundannahme, an der Theorien verborgener Variablen scheitern, die Lokalität ist.--Claude J (Diskussion) 08:18, 19. Apr. 2018 (CEST)
Danke für Eure Erklärungen. Da es tatsächlich nicht um ein Spezifikum der Variablen selbst geht, sondern um den zu Grunde liegenden theoretisschen Standpunkt, könnte ich sagen, dass er durch den Nachsatz doch klar benannt wird: Einstein verteidigt den lokal-realistischen Standpunkt. Für mehr Genauigkeit schlage ich vor: „Einsteins Konzept der (lokalen) verborgenen Variablen“. Die Klammern deuten dann an, dass das zwar nicht bei Einstein steht, dass es aber um eben diesen Gegenstand geht. OK?- Übrigens, vielleicht muss ich da nochmal nachlesen, aber könnte da nicht auch der Artikel Verborgene Variablen deutlicher werden?-- Binse (Diskussion) 12:42, 19. Apr. 2018 (CEST)
Die Benennung stammt so viel ich weiss von von Neumann (verborgene Parameter), wobei er den Vergleich mit der statistischen Mechanik zog (die ist auch durch "verborgene" Variablen der klassischen Mechanik erklärt worden). Einstein meidet den Begriff in dem EPR-Aufsatz so weit ich sehe, sie gebrauchen den Begriff der "Vollständigkeit". Sie wollen nach eigenen Worten zeigen dass die qm Beschreibung mit einer Wellenfunktion in einer realistischen Theorie unvollständig ist. Meiner Kenntnis nach hat Einstein auch keinen detaillierten Entwurf einer alternativen Interpretation vorgelegt.--Claude J (Diskussion) 13:43, 19. Apr. 2018 (CEST)
Mir scheint, nachdem seit einer Woche keine weiteren Anregungen mehr eingegangen sind, sollte ich jetzt mal die Einleitung auswechseln. Ich habe den Entwurf noch einmal überarbeitet und insbesondere die verborgenen Variablen herausgenommen, da Einstein diese Bezeichnung offenbar nicht benutzt hat. Wer es für wichtig hält, kann eine Bemerkung einfügen, dass diese Eigenschaften später so genannt wurden. Meine gesammelte Kritik an der gegenwärtigen Fassung der Einleitung schreibe ich in den nächsten Abschnitt von Claude J.-- Binse (Diskussion) 12:33, 27. Apr. 2018 (CEST)

Zum Abschnitt „Realismus und Lokalität“

Im Abschnitt „Realismus und Lokalität“ ist mehr falsch als richtig. Wäre schön, wenn sich wer, der über Quellen verfügt, der Sache mal annehmen könnte.

Im ersten Absatz ist mir unverständlich: „Wenn also das Ergebnis jeder denkbaren Messung feststeht, auch wenn es wegen ungenügender Kenntnis verborgener Parameter nicht vorher bekannt ist“. Realismus oder nicht, das Ergebnis ist vor der Messung in aller Regel nicht bekannt. Wozu messe ich sonst? Was hat das zu tun mit ungenügender Kenntnis verborgener Variablen?

Im zweiten Absatz gibt es gleich einen ganzen Haufen Fehler. Für mein Gefühl ist fast alles falsch.

Zunächst wäre das, was angeführt ist (wenn es denn richtig wäre) doch höchstens ein Beispiel und nicht das Wesen von Nichtlokalität, wie die Formulierung verspricht. Ich denke da z.B. an den Welle-Teilchen-Dualismus, den man jedenfalls als nichtlokalen Effekt (mit nur einem Teilchen!) ansehen kann.

Zweitens haben nicht zwei Teilchen einen „raumartigen Abstand“, sollte besser heißen: „stehen in raumartiger Beziehung“, sondern zwei Ereignisse, in unserem Fall zwei Messungen.

Drittens gehört die raumartige Beziehung nicht zum Wesen eines nichtlokalen Effekts, sondern wird in Experimenten, die Nichtlokalität zeigen sollen, eigens gewählt, um klassische Kausalketten auszuschließen. (Wenn wir uns einig sind, dass wir eh nur ein Beispiel ansprechen, ist das ok).

Viertens bestreite ich vehement, dass sich die Messungen in den bekannten Experimenten überhaupt beeinflussen. Die Korrelationen, die die Theorie vorhersagt, und die von Experimenten bestätigt werden, bedeuten oder beweisen keineswegs, dass eine Messung die andere beeinflusst. Leider kann ich hiergegen nichts ausrichten, denn es gibt genug Zitiermöglichkeiten für diesen Aberglauben. Manche "Fachleute" schreiben leider so daher nach dem Motto: „Versteht der Laie eh nicht“.

Fünftens aber: „...wenn sich die Wahl dessen, was beim einen Teilchen gemessen wird, bei der Messung nicht unmittelbar auf das andere Teilchen auswirkt“, ist für mich logisch sinnlos. Die Wahl an sich, also ohne Messung, die bloße Absicht, hat ohnehin keine Auswirkung. Und eine durchgeführte Messung setzt doch selbstverständlich voraus, dass man erst bestimmt hat, was gemessen werden soll, ob z.B. die up/down-Komponente oder stattdessen die left/right-Komponente des Spins. Wenn also überhaupt die Messung einen Einfluss auf das andere Teilchen hätte (was ich, wie gesagt, bestreite) ist es überflüssig und entschieden verwirrend, eigens die Wahl der zu messenden Größe zu erwähnen. Die gehört einfach zu der betreffenden Messung.

Wenn dann weiter unten im Artikel die Möglichkeit einer solchen Wahl, d.h. wie mir scheint, die Möglichkeit, bestimmte Variablen zu messen, in Frage gestellt wird, dann müsste dazu entschieden mehr gesagt werden.-- Binse (Diskussion) 22:54, 19. Jul. 2017 (CEST)

1. Darauf habe ich ja schon vor einem Jahr versucht aufmerksam zu machen: Der Satzteil
...oder dass man nicht beliebig wählen kann, welche Eigenschaft des Systems gemessen werden soll.
ist ein ausgesprochener Fremdkörper im Text. Zu den zwei Themen der Überschrift kommt hier unvermittelt ein drittes. Und dieses, denke ich, versteht keine Sau. Was mag da überhaupt gemeint sein? Wer oder was sollte mich hindern, zu wählen, welchen von drei Knöpfen ich drücke? Genauso gut könnte man dazu schreiben: “Oder, dass der Experimentator nicht nachdenken kann”. Ich streiche das jetzt mal, und wer meint, dass es doch hingehört, kannn es nach unseren Regeln ja wieder hin tun. Dann aber bitte mit soviel Erklärung, dass es Sinn gibt.
2. Habe ich auch schon angesprochen, hat leider nichts bewirkt. Die Theorie sagt klar und Experimente bestätigen, dass die Messergebnisse an verschränkten Teilchen nichtlokal korreliert sind, d.h. über Entfernungen, die im Zusammenhang der Messungen nicht mit Signalen überbrückt werden können. Dies zu akzeptieren bedeutet, dass die Welt nach heutigem Wissen nicht durch eine lokale (und realistische) Theorie beschrieben werden kann. Das muss also im Artikel stehen. Ein Einfluss einer Messung auf eine entfernte andere Messung ist für die Argumentation dagegen nicht wichtig, ist bei näherem Hinsehen nicht gegeben und sollte daher auch nicht behauptet werden. Alle anderen Wissenschaftler, die mit Wahrscheinlichkeiten zu tun haben, (Mediziner, Volkswirtschaftler etc.) sind nach meiner Beobachtung ganz konsequent darin, für jede beobachtete Korrelation sorgfältig nach einem etwaigen Kausalzusammenhang zu suchen, statt ihn einfach zu behaupten. Einzig die Quantentheoretiker nehmen sich heraus, eine Korrelation einfach so als (Fern-)Wirkung oder als Einfluss zu beschreiben. Dass sie dass nie irgendwie begründen, erweckt zumindest den Einruck, dass sie sich der logischen Lücke in der Argumentation gar nicht bewusst sind. Ich formuliere den Abschnitt mal so, dass er auf Korrelation statt auf Einfluss Bezug nimmt. Wer das dann zurückdrehen will, kann das natürlich tun, ist aber herzlich um eine überzeugende Begründung gebeten. Eventuell lässt sich ja auch erst diskutieren, wozu haben wir diese Seite, und dann vielleicht wieder ändern.–– Binse (Diskussion) 00:28, 5. Jul. 2019 (CEST)

Umarbeitung

Wie angekündigt habe ich in einem ersten Schritt den Beweis der Ungleichung durch den viel verständlicheren Beweis ersetzt, den ich in Zeilingers Buch „Einsteins Spuk“ gefunden habe. Dort ist er allerdings grausam weitschweifig. Als Quelle gebe ich die Arbeit von Wigner an, die Zeilinger nennt. Da ich leider keine Antwort auf meine Frage im vorigen Abschnitt bekommen habe, lege ich den Singulettzustand zugrunde. Mit dem, den ich oben Märchenzustand nannte, wäre es einfacher. Leider kann ich den Singulettzustand nicht verlinken, weil dort nichts steht was hier hilft.

Ok! Kein Märchen, den Zustand gibt es. Ich habe meinen Rechenfehler gefunden. mit für horizontal/vertikal ist invariant gegen Veränderung der Polarisationsrichtung. Darstellumg im Artikel entsprechend vereinfacht.– Binse (Diskussion) 18:10, 14. Aug. 2019 (CEST)

Als nächstes soll eine andere Beschreibung der quantentheoretischen Sicht folgen; die finde ich, so wie sie ist, höchst unbefriedigend.– Binse (Diskussion) 01:06, 27. Jul. 2019 (CEST)

Der Abschnitt muss ja schon wegen der Bezüge auf den jetzt gelöschten Abschnitt ersetzt werden.– Binse (Diskussion) 15:43, 27. Jul. 2019 (CEST)

Neuer schlupflochfreier Test

Laut dem Artikel „Endgültiges Aus für eine anschauliche Physik“ vom 9.9.2015 im Abschnitt „Forschung und Technik“ der Neuen Zürcher Zeitung liegt jetzt erstmals ein Experiment vor, das im Gegensatz zu den bisherigen Widerlegungsexperimenten keine Ausflüchte bezüglich „kleiner“ Schlupflöcher mehr zulässt. Wenn das stimmt, dann sollte das Experiment der internationalen Forschergruppe unter Leitung von Ronald Hanson von der Delft University of Technology mit verschränkten Spins von Elektronen im Abschnitt „Widerlegungsexperimente“ aufgenommen werden. --Dokkalli (Diskussion) (19:30, 12. Dez. 2015 (CET), Datum/Uhrzeit nachträglich eingefügt, siehe Hilfe:Signatur)

Dazu gibt es nun die Referenz 13 auf die Arbeit von Hanson et al. Im Text wird die Arbeit (so weit ich es beurteilen kann) korrekt beschrieben. Der Punkt ist damit aus meiner Sicht erledigt. --B wik (Diskussion) 23:11, 7. Jun. 2020 (CEST)

Verschränkte Photonen

Im Artikel klingt es so, als seien zwei Photonen automatisch verschränkt, wenn sie nur von der gleichen Quelle kommen. Und wenn sie verschränkt sind, haben sie angeblich die gleiche Polarisationsebene. Soll man das glauben?-- Binse (Diskussion) 22:43, 14. Jul. 2013 (CEST)

Text wurde mittlerweile entsprechend abgeändert. --B wik (Diskussion) 08:13, 24. Sep. 2020 (CEST)

Beschreibung nicht konsequent durchgängig

In der Herleitung wird zur Verdeutlichung der Polarisationsmessung die Ebenen a und b genannt, welche senkrecht zueinander stehen. Das ist auch einleuchtend. Wenn es danach aber zu Man kann sich nun eine Reihe von wiederholten Messungen an Photonenpaaren vorstellen kommt, dann wird eine Polarisatiopnsebene c eingeführt, die zuvor nirgendwo erwähnt wird und die in keinem Bezug zu dem steht, was davor beschrieben wurde. Ohne eine Erläuterung die erklärt, in welchem Zusammenhang a, b und c jeweils zueinander stehen, ist die gesamte Erklärung nahezu nutzlos. Chiron McAnndra (Diskussion) 16:11, 12. Apr. 2016 (CEST)

Der Übergang von der bellschen Ungleichung konkreter Meßergebnisse hin zur Einbringung der Wahrscheinlichkeit von Messergebnissen der klassischen Physik für die nachfolgende Herleitung des Widerspruches ist abrupt und bleibt ohne Erklärung. Spätestens hier kommt der Laie ins Grübeln.

Text wurde mittlerweile entsprechend angepasst. --B wik (Diskussion) 08:15, 24. Sep. 2020 (CEST)

Abschnitt 2 (Die Ungleichung bei Annahme von verborgenen Variablen)

Einiges an diesem Abschnitt halte ich für fragwürdig:

  1. Ist es sinnvoll, eine Überschrift mit einem Literaturverweis zu versehen? Sie enthält ja keine Behauptung, die es zu belegen gäbe.
  2. Entspricht ein Pluralis auctoris wie „sagen wir“ enzyklopädischem Stil? Man könnte ja auch beispielsweise schreiben.
  3. Ist es in Ordnung, dass in ein und demselben Abschnitt bestimmte Ausdrücke „mathematisch“ () geschrieben werden, die mit α und β notierten Komponenten aber nicht?

Bei (3) habe ich keine Ahnung, bei (1) und (2) würde ich eine negative Antwort vorschlagen. --GALTZAILE PPD () 19:28, 7. Okt. 2019 (CEST)

Habe Punkt 1 erledigt. Eine Referenz in der Überschrift erscheint mir unsinnig, bzw. ungebräuchlich zu sein. Punkt 2 ist eine Stilfrage und damit für mich in der Priorität nicht wirklich hoch gesetzt. Da gibt es meiner Ansicht nach momentan viel dringendere Baustellen. --B wik (Diskussion) 09:10, 23. Sep. 2020 (CEST)

Hilfe erbeten

Entschuldigt, mir gehts gut, dem Artikel weniger. Während ich versuche, ihn zu renovieren, stoße ich auf zwei Probleme, von denen ich hoffe, von den Fachkundigen unter uns unterstützt zu werden. Ausgangspunkt sind für mich der aktuelle Artikel und die Darstellung in Zeilingers: Einsteins Spuk, dessen kombinatorischen Beweis der Ungleichung ich für WP deutlich geeigneter finde.

1. Der Beweis in unserem Artikel verwendet einen Zustand des Photonenpaars, in dem für jede Orientierung der Polarisationsfilter die beiden Photonen gleich reagieren, also beide das Filter passieren oder beide absorbiert werden. Da ich meine, dass es diesen Zustand des Paares nur im Märchen gibt, habe ich kürzlich begonnen, unseren Text so umzuschreiben, dass er stattdessen den Singulettzustand verwendet, in dem die beiden Photonen stets entgegengesetzt reagieren. Nun bemerke ich aber zu meinem Schrecken, dass auch Zeilinger diesen ‚Märchenzustand‘ verwendet. Meine Frage, daher: Gibt es ihn, oder nicht?

2. Allgemein heißt es ja, wegen der Verletzung der bellschen Ungleichung müsse man das Relitätsprinzip oder das Lokalitätsprinzip aufgeben. Beide sind also offenbar hinreichende Bedingungen im Beweis der Ungleichung. Da der Artikel immerhin eine Herleitung bietet, die dafür ja auch nicht zu schwierig ist, muss der Beweis aber auch auf die Punkte hinweisen, wo die Bedingungen gebraucht werden. Mit dem Realitätsprinzip tut er das, für das Lokalitätsprinzip aber nicht. Bei Zeilinger fand ich keinen entsprechenden Hinweis und selbst kann ich auch nicht sehen, wo denn eigentlich Lokalität notwendig ist. Das ist also der andere Punkt, an dem ich um sachkundige Hilfe bitte.–– Binse (Diskussion) 18:14, 7. Jul. 2019 (CEST)

zu 1.
Bell verwendet im Originalpaper den Singulettzustand. Dort haben die beiden Teilchen bei einer Messung grundsätzlich entgegengesetzte Eigenschaften. Im Abschnitt "Die Ungleichung bei Annahme von verborgenen Variablen" sind die beschriebenen Überlegungen so allgemein formuliert, dass es nicht unmittelbar darauf ankommt, ob die Eigenschaften gleich oder entgegengesetzt sind. AZ wird in seinem Buch wohl ähnliche Überlegungen verwenden. Leider habe ich sein Buch aber selbst noch nicht gelesen. --B wik (Diskussion) 21:25, 23. Sep. 2020 (CEST)
Korrektur: Ich denke, den Zustand gibt es schon. Man beginnt mit einem Paar aus einem links- und einem rechtszirkular polarisierten Photon und hat damit die Spins korrekt. Der benötigte Zustand ist dann eine Superposition aus dem beschriebenen Zustand und einem entsprechend rechts- und linkszirkular polarisierten Paar. So erhält man zwei Photonen, welche die zwei Filter in der gleichen Richtung passieren können. --B wik (Diskussion) 21:01, 10. Nov. 2020 (CET)

Lemma-Schreibweise

Argumente gibt es hier: https://de.wikipedia.org/w/index.php?title=Bellsche_Ungleichung&curid=200194&diff=204898512&oldid=204885539 Einschätzungen erbeten. Gibt es nicht Beispiele, wie bisher mit sowas umgegangen wird? --Zulu55 (Diskussion) 09:24, 26. Okt. 2020 (CET)

Es sollte auf jeden Fall im Artikel einheitlich sein. Aus Lehrbüchern ist mir eher der Schreibweise ohne Apostroph geläufig. Ansonsten fällt mir noch das Beispiel ein. --Doc ζ 21:23, 26. Okt. 2020 (CET)

Strahlteiler sind keine Filter

Das Wort ‚Filter‘ ist zwar kürzer als ‚Strahlteiler’. Rechtfertigt das den ständigen falschen Sprachgebrauch im Artikel? Wie andere Filter auch hält ein Polarisationsfilter das was unerwünscht ist zurück, absorbiert es. Text und Skizze machen ja auch klar, dass Strahlteiler gemeint sind. Ich empfinde diese Sprache als fehlerhaft. Naja.– Binse (Diskussion) 23:14, 18. Nov. 2020 (CET)

Abweichend von der bellschen Originalarbeit funktioniert die gesamte Argumentation des Artikels meiner Meinung nach durchgängig nur mit zwei Polarisationsfiltern. Nur diese kann man um einen Winkel so drehen, dass die Rechnungen im Artikel auch Sinn machen. Entsprechende Versuchsbeschreibungen findet man auch in den zugehörigen aktuellen Veröffentlichungen. Strahlteiler werden meiner Meinung nach im Artikel nicht benötigt. --B wik (Diskussion) 21:57, 22. Nov. 2020 (CET)
In der Arbeit von Zeilinger et al. aus dem Jahr 2015 https://arxiv.org/abs/1511.03190 mit Beschreibung des Versuchsaufbaues werden einstellbare Polarisationsfilter verwendet. Ob dort auch reflektierte oder nur transmittierte Photonen gemessen werden, kann ich der Arbeit aktuell leider auch nicht entnehmen. Zur Not nimmt man als Wahrscheinlichkeit für Reflektion einfach die Wahrscheinlichkeit 1 - P(Transmission). --B wik (Diskussion) 20:35, 24. Nov. 2020 (CET)
Drehen kann man Filter mit der notwendigen Geschwindigkeit genau so wenig, wie Strahlteiler. Tatsächlich werden stattdessen die Schwingungsebenen der Photonen in Gegenrichtung um den gewünschten Winkel gedreht, indem man sie durch eine Kerr- oder Pockelszelle schickt, bei der man den Drehwinkel durch die angelegte Spannung sehr schnell ändern kann. Welche der beiden Typen tatsächlich verwendet wird, weiß ich leider nicht. Ist hier ja nicht wichtig.
Dass das Experiment eigentlich keine Strahlteiler braucht, ist Theorie. Da die praktisch vorhandenen Detektoren einen beträchtlichen Prozentsatz der Photonen, die sie erreichen, fälschlich nicht anzeigen, weiß man bei Verwendung von Filtern nicht, ob ein fehlendes Photon im Filter absorbiert (sagen wir vertikal polarisiert) wurde, oder durchgelassen (horizontal p.) und nur nicht detektiert. Bei Verwendung von Strahlteilern stört das wenig, weil man einfach die Ereignisse ignoriert, bei denen keiner der Detektoren anschlägt.
Im Übrigen zeigt ja auch das Diagramm Strahlteiler.– Binse (Diskussion) 17:19, 3. Dez. 2020 (CET)
Die Quanteneffizienz der Nachweisgeräte führt (angeblich) zu einer Art Nachweis-Schlupfloch, das aber (angeblich) geschlossen werden kann. Vielleicht muss da nur ein konstanter Multiplikator berücksichtigt werden? Wie die bellsche Ungleichung mit Strahlteilern überprüft werden kann, weiß ich leider nicht. Habe dazu keine verwertbaren Ideen. --B wik (Diskussion) 22:26, 6. Dez. 2020 (CET)

Quantentheoretische Wahrscheinlichkeiten

Im aktuellen Artikel wird die Wahrscheinlichkeit W(ah,bh) mit 75% angegeben. Auch W(bh,ch) wäre demnach 75%. Heißt das nun tatsächlich: Experimentell werden in 75% der Fälle, in denen beide Photonen registriert werden, während der eine Filter auf Position A und der andere auf Position B steht, beide Photonen als passiert registriert? Genauso in 75% der Fälle, in denen der eine Filter auf Position B und der andere auf Position C steht? (Es ist ersichtlich, dass dies aus dem Gesetz von Malus für Photonen folgt, welche die Quelle mit Polarisationsrichtung B verlassen haben. Ist es aber auch der Fall für Licht, welches die Quelle unpolarisiert verlässt?) (nicht signierter Beitrag von ZukunftHatBegonnen (Diskussion | Beiträge) 15:27, 8. Jul. 2020 (CEST))

Kann diese Angaben im Artikel nicht finden. Hat sich damit wohl erledigt. --B wik (Diskussion) 21:03, 10. Nov. 2020 (CET)

Die Frage bezieht sich auf den Abschnitt "Verletzung der Ungleichung in der Quantentheorie". Dort heißt es, gegen Ende des Abschnitts: "Wählt man ∠ ( a , b ) = ∠ ( b , c ) = π / 6 = 30 ∘ [...], so ergäbe sich 3/4 ≤ 1/4 + 1/4 , [...] was offenbar falsch ist." Dies klingt so, als sei die Wahrscheinlichkeit, dass (im Falle von Licht, welches die Quelle unpolarisiert verlässt) in 75% der Fälle Photonen hinter beiden Filtern registriert werden, wenn der eine Filter auf A und der andere auf B steht; oder auch, wenn der eine Filter auf B und der andere auf C steht. (nicht signierter Beitrag von ZukunftNeu (Diskussion | Beiträge) 13:03, 13. Nov. 2020 (CET))

OK. Das Experiment wird zwar mehrfach, aber immer nur mit einem verschränkten Photonenpaar durchgeführt. Bei diesem quantenmechanischen Zwei-Teilchen-Zustand hat man dann AFAIK zwei identische Polarisationszustände bezüglich eines Polarisationsfilters.
Ich möchte deine Frage:

Experimentell werden in 75% der Fälle, in denen beide Photonen registriert werden, während der eine Filter auf Position A und der andere auf Position B steht, beide Photonen als passiert registriert? Genauso in 75% der Fälle, in denen der eine Filter auf Position B und der andere auf Position C steht?

also mit ja beantworten.
Das Gesetz von Malus habe ich im Abschnitt als (eher themenfremde) Randnotiz belassen. --B wik (Diskussion) 20:38, 16. Nov. 2020 (CET)
Entschuldigung, aber ich versuche nochmal genauer zu formulieren, was ich meine. Du schreibst: "Das Experiment wird zwar mehrfach, aber immer nur mit einem verschränkten Photonenpaar durchgeführt. Bei diesem quantenmechanischen Zwei-Teilchen-Zustand hat man dann AFAIK zwei identische Polarisationszustände bezüglich eines Polarisationsfilters." Eingängig ist, dass ein Photonenpaar mit der (identischen) Polarisationsrichtung A oder B bei der Filterstellung A und B (bei ∠ ( A , B ) = 30 ∘) zu 75% Registrierung hinter beiden Filtern führt. Ebenso ist plausibel, dass ein Photonenpaar mit der Polarisationsrichtung B oder C bei der Filterstellung B und C (bei ∠ ( B , C ) = 30 ∘) zu 75% Registrierung hinter beiden Filtern führt. Auch ist ersichtlich, dass eine Polarisationsrichtung von A oder C (bei ∠ ( A , C ) = 60 ∘) 25% ergäbe. Doch hier wechselt ja die Polarisation des Photonenpaars. (In den ersten beiden Fällen könnte sie noch konstant auf B sein, nicht jedoch mehr im letzten Fall.) Vielleicht missverstehe ich ja etwas am Aufbau oder den Erwartungswerten, aber mir scheint, die hier angegebenen Wahrscheinlichkeiten würden nur für bestimmte Fälle gelten, die nicht miteinander vereinbar sind. --ZukunftNeu (Diskussion) 01:36, 16. Jan. 2021 (CET)
Die "echte" Polarisationsrichtung des Lichtes kennt man nicht und es ist nichtmal klar, ob es so etwas wie eine "echte" Polarisationsrichtung des Lichtes gibt.
Ja, in 25% der Fälle werden beide verschränkten Photonen detektiert, wenn das eine Photon die Polarisationsrichtung A hat und das andere dazu verschränkte Photon die Polarisationsrichtung C aufweist. Ob die beiden Photonen auch zusätzlich die Polarisationsrichtung B haben/hätten? Das weiß man nicht. Das wurde in diesem Fall nicht gemessen. --Eulenspiegel1 (Diskussion) 03:07, 16. Jan. 2021 (CET)
Danke für die Antwort. Mir geht es um den Abschnitt "Verletzung der Ungleichung in der Quantentheorie". In diesem Abschnitt werden die (von mir zitierten) Filtereinstellungen A, B und C als Beispiel herangezogen. Mir scheint, dass es sich bei den angegebenen Wahrscheinlichkeiten um bedingte Wahrscheinlichkeiten handelt. 1.) Wenn eine Quelle Photonenpaare mit bestimmten Eigenschaften erzeugt, dann werden bei der Filterstellung A und B 75% der Photonen detektiert. 2.) Wenn eine Quelle Photonenpaare mit bestimmten Eigenschaften erzeugt, dann werden bei der Filterstellung B und C 75% der Photonen detektiert. 3.) Wenn eine Quelle Photonenpaare mit bestimmten Eigenschaften erzeugt, dann werden bei der Filterstellung A und C 25% der Photonen detektiert. Meine Frage ist nun: Kann ein und dieselbe Quelle wirklich Photonenpaare hervorbringen, die alle drei Punkte erfüllen? (Das wäre bspw. dann der Fall, wenn die Filter dazu führen würden, dass in 100% der Fälle das eine Photon vom ersten Filter in die Richtung dieses Filters polarisiert wird, dieses erste Photon in 100% der Fälle registriert wird und sich diese Polarisation auf das zweite Photon überträgt, das dann in 75% der Fälle bzw. in 25% der Fälle registriert wird. Ein Szenario, das nicht gerade plausibel klingt. Also meine Frage ist: gibt es tatsächlich eine Photonenquelle bzw. -verschränkung, für die alle die im Artikel genannten und von mir zitierten Wahrscheinlichkeiten zutreffen?) --ZukunftNeu (Diskussion) 14:17, 16. Jan. 2021 (CET)
Ja, man hat eine einzige Quelle, die sich nicht ändert. Diese Quelle sendet zufällig verschränktes Licht. Ob die Filter A,B oder A,C oder B,C filtern, wird zufällig bestimmt. Und zwar erst zufällig, nachdem das Photonenpaar bereits erzeugt wurde.
Ja, das klingt extrem unplausibel. So unplausibel, dass viele Wissenschaftler der Meinung waren, mit der Bellschen Ungleichung könnte man die Quantentheorie widerlegen. Sie waren dann sehr überrascht, dass die Bellsche Ungleichung die Quantentheorie bestätigt hat. Die Ergebnisse des Experiments zeigen letztendlich, dass manchmal das Plausible falsch ist und das Unplausible richtig ist. --Eulenspiegel1 (Diskussion) 14:33, 16. Jan. 2021 (CET)
Danke für die Antwort. Heißt das, die im Artikel angegebenen Wahrscheinlichkeiten gelten tatsächlich für in zufälliger Richtung polarisierte (verschränkte) Photonen? Dies würde bedeuten, dass bei Filterstellungen A und B (oder auch bei Filterstellungen B und C) die Wahrscheinlichkeit, beide Photonen zu detektieren, höher wäre, als die Wahrscheinlichkeit, ein einzelnes (unverschränktes) Photon zufälliger Polarisierung mit einem Filter zufälliger Ausrichtung zu detektieren! (Die Wahrscheinlichkeiten für die Detektion beider Photonen für die ersten beiden Fälle wird im Artikel mit 75% angegeben. Die Wahrscheinlichkeit, ein einzelnes Photon bei zufälliger Filterstellung zu detektieren, sollte 50% sein.) Entspricht dies tatsächlich den empirischen Beobachtungen? -- ZukunftNeu (Diskussion) 16:58, 16. Jan. 2021 (CET)
Nein, das müsste man im Artikel noch genauer erklären:
Das ist die Wahrscheinlichkeit, dass der zweite Filter das Photon detektiert, wenn der erste Filter das Photon detektiert.
Das heißt die Filtereinstellung A,A würde 100% ergeben. A,B ergibt 75%. A,C ergibt 25%. Und A,D ergäbe 0%. (Wenn D um 90° um A gedreht wäre.)
Wenn man die Gesamtwahrscheinlichkeit wissen will, müsste man das ganze mit 50% multiplizieren.
Das heißt die Filtereinstellung A,A würde 50% ergeben. A,B ergibt 37,5%. A,C ergibt 12,5%. Und A,D ergäbe 0%. (Wenn D um 90° um A gedreht wäre.) --Eulenspiegel1 (Diskussion) 17:35, 16. Jan. 2021 (CET)
OK, danke für den schließlich produktiven Austausch. Im Artikel klingt es so, als handle es sich um absolute Wahrscheinlichkeiten. Tatsächlich werden aber demnach hier bedingte Wahrscheinlichkeiten angegeben. (Dann müsste auch deutlicher gemacht werden, auf welcher Basis es gerechtfertigt ist, diese bedingten Wahrscheinlichkeiten in einer Ungleichung zusammenzuführen.)--ZukunftNeu (Diskussion) 18:02, 16. Jan. 2021 (CET)
Ich habe im Artikel gerade die bedingten Wahrscheinlichkeiten durch absolute Wahrscheinlichkeiten ersetzt. --Eulenspiegel1 (Diskussion) 18:30, 16. Jan. 2021 (CET)
Jetzt stimmt aber die Wahrscheinlichkeit für W (bh,cv) im Artikel nicht mehr, oder? Die wäre jetzt 1-0,5*cos²(bh,ch) oder? --ZukunftNeu (Diskussion) 18:54, 16. Jan. 2021 (CET)
Nein, die absolute Wahrscheinlichkeit für W (bh,cv) ist: 0,5*(1-cos²(bh,ch))= 0,5*sin²(bh,ch). --Eulenspiegel1 (Diskussion) 18:57, 16. Jan. 2021 (CET)
OK, das stimmt, danke. --ZukunftNeu (Diskussion) 19:08, 16. Jan. 2021 (CET)

„Unterschiedliche Filter“

Mit diesem Edit habe ich versucht, darzustellen, dass es sich nicht um unterschiedliche Filter handelt (also Filter mit unterschiedlichen Eigenschaften), sondern dass die beiden Filter so identisch wie möglich und damit optimalerweise (bis auf deren Position natürlich) ununterscheidbar sein sollen. Zusätzlich habe ich den Aspekt erwähnt, dass das Eintreffen der Photonen auf den beiden Filtern möglichst gleichzeitig sein soll, damit klar ist, dass beiden Messungen sich nicht beeinflussen können (man hätte hier etwas von „raumartigem Abstand“ schreiben können, das erschien mir an dieser Stelle übertrieben). Und ich habe die „Messrichtungen a, b, c“ in „a, b oder c“ umformuliert, damit klar ist, dass bei jeder Messung immer genau eine Richtung gemessen wird.
Eulenspiegel1 hat meine Formulierung offenbar so verstanden, dass ich der Ansicht wäre, beide Photonen würden auf den gleichen Filter treffen („Nein, der eine Filter steht links und der andere Filter steht rechts. Der Clou ist ja, dass es eben nicht der selbe Filter ist, sondern zwei Filter, die weit genug voneinander entfernt stehen.“), was ich nicht wirklich nachvollziehen kann. Kann ein gutgläubiger Leser meine Formulierung „Beide Photonen treffen gleichzeitig auf Polarisationsfilter, die unabhängig voneinander die Messrichtungen a, b oder c haben können“ wirklich missverstehen als „es gibt nur einen Filter, auf den beide Photonen treffen“? Besteht nicht eher das Risiko, die aktuelle Formulierung „Beide Photonen erreichen jeweils einen unterschiedlichen Filter, der jeweils die Messrichtungen a, b, c haben kann mit den beiden „jeweils“ dahingehend misszuverstehen, dass beide Photonen auf beide Filter treffen oder/und diese Messrichtungen alle gleichzeitig geprüft werden?
Ich halte die aktuelle Formulierung nicht für so perfekt, dass eine Verbesserung nicht mehr möglich ist, und ich halte meine Formulierung insgesamt für verständlicher (und für inhaltlich mindestens genauso richtig wie den aktuellen Text). Ich finde die vollständige Rücksetzung (noch dazu mit einem Kommentar, der mich hinstellt, als hätte ich die elementarsten Basics nicht verstanden), für nicht gerechtfertigt und enzyklopädisch nicht hilfreich und bitte um Meinungen.
Troubled @sset   [ Talk ]   10:28, 17. Jan. 2021 (CET)

Gegenfrage: Kann ein gutgläubiger Mensch wirklich missverstehen, dass die beiden Filter unterschiedliche Eigenschaften haben?
Aber um deine Frage zu beantworten: Ja, normalerweise sind die Photonen-Experimente so aufgebaut, dass das Photon erst durch den einen Filter geht und anschließend durch einen zweiten Filter, der dahinter steht. Die Besonderheit an diesem Experiment ist, dass die beiden Photonen eben nicht nacheinander die gleichen Filter passieren, sondern jeweils unterschiedliche Filter passieren. Das ist das, was dieses Experiment von anderen ähnlichen Experimenten unterscheidet. --Eulenspiegel1 (Diskussion) 21:54, 17. Jan. 2021 (CET)
Sie passieren eben nicht unterschiedliche Filter, sondern identische Filter. Die stehen an verschiedenen Orten und können unterschiedlich orientiert sein, aber das macht sie nicht zu unterschiedlichen Filtern.
Autos werden normalerweise mit vier identischen Reifen bestückt. Würdest du trotzdem behaupten, dass dein Auto mit vier unterschiedlichen Reifen fährt, nur weil die an unterschiedlichen Stellen am Fahrzeug montiert sind?
Ich nehme aber zur Kenntnis, dass unsere sprachlichen Vorstellungen von „korrekt, verständlich und sinnvoll“ diametral auseinanderliegen. Troubled @sset   [ Talk ]   18:19, 18. Jan. 2021 (CET)
Die beiden Filter stehen an verschiedenen Orten, deswegen passieren sie eben nicht die selben, sondern unterschiedliche Filter. Dass die beiden Filter identisch sind, ist trivial. Das muss man nicht extra erwähnen. Dass es sich nicht um die selben Filter handelt, ist jedoch nicht trivial und muss erwähnt werden.
Ich würde sagen: "Mein Auto fährt mit 4 Reifen." Aber wenn jemand zweimal auf meinen Reifen schießt, würde ich entweder sagen: "Er schoss zweimal und traf den selben Reifen." oder "Er schoss zweimal und traf jeweils einen unterschiedlichen Reifen."
Du machst den Fehler und siehst "unterschiedlich" als Gegensatz zu "identisch". Das ist jedoch falsch. Die Gegensatzpaare sind: "identisch <-> nicht-identisch" und "der Selbe <-> unterschiedliche".
Aber wir können auch gerne "verschiedene" anstatt "unterschiedliche" schreiben, falls dir das Wort besser gefällt. --Eulenspiegel1 (Diskussion) 19:27, 18. Jan. 2021 (CET)
Dass diese Gegensatzpaare von der großen Mehrheit so gesehen werden wie von dir, könnte man durchaus hinterfragen. Ich würde das eher so verstehen: „derselbe“ <–> „nicht derselbe/verschiedene“ und „identisch“ <–> „unterschiedlich“.
„Unterschiedliche Maßnahmen“ unterscheiden (!) sich inhaltlich; wenn die gleiche Maßnahme an mehr als einer Stelle angewendet wird, ist das immer noch die gleiche Maßnahme. Wenn du zwei Eimer Farbe mit identischem Inhalt nimmst und damit zwei Wände anmalst, dann sind das nicht unterschiedliche Farben, nur weil sie zu verschiedenen Zeiten oder/und an verschiedenen Orten aufgebracht wurden. Und es sind auch nicht „unterschiedliche“ Wände, nur weil es sich um verschiedene Wände handelt. „Die Farbe ist für unterschiedliche Wände geeignet“ heißt, dass sie auf unterschiedlichem Grund aufgebracht werden kann, nicht dass sie an mehr als einer Wand benutzt werden kann.
„Er schoss zweimal und traf jeweils einen unterschiedlichen Reifen“ würde ich auch nicht so formulieren. „… und traf zwei Reifen“, würde ich schreiben, oder, wenn man klarstellen will, dass nicht mit einem der beiden Schüsse zwei Reifen getroffen wurden, „… und traf jeweils einen Reifen“. Niemand würde hier annehmen, dass zwei Mal der gleiche Reifen getroffen wurde. Um auch das noch auszuschließen (mMn unnötig), würde ich schreiben „… und traf jeweils einen anderen Reifen“. Welche zusätzliche, hilfreiche und notwendige Information bekommt man hier durch den Einbau von „unterschiedlich“?
„Unterschiedliche Filter“ sind für mich IR-Filter, UV-Filter, Pol-Filter, Grau-Filter etc. In diesem Experiment hier sind die Filter identisch, nicht unterschiedlich, und die Verwendung dieses Worts ist nicht nur unnötig, sondern verwirrend.
Ich denke, dass „unterschiedlich“ oder auch „verschieden“ gar nicht nötig sind, wenn einmal klar ist, dass es sich um je einen Filter für jedes Photon handelt. Hier noch ein anderer Formulierungsvorschlag von mir:
Die beiden Photonen treffen auf je einen Filter; die Filter sind unabhängig voneinander auf die Messrichtung a, b oder c eingestellt.
Troubled @sset   [ Talk ]   15:00, 19. Jan. 2021 (CET)
In Ordnung. Dann schreiben wir es so wie in deinem letzten Vorschlag. --Eulenspiegel1 (Diskussion) 18:41, 19. Jan. 2021 (CET)

Wurde die Quantenmechanik damit bewiesen?

Die Vorhersagen der Quantenmechanik zu bestimmten Experimenten widersprechen der bellschen Ungleichung und die experimentellen Ergebnisse tun es ebenfalls. Aus dem lokalen Realismus folgt, dass die bellsche Ungleichung immer erfüllt sein muss. Diese Vorhersage widerspricht vielen experimentellen Ergebnissen eindeutig. Der lokale Realismus ist damit experimentell widerlegt. Über die Quantenmechanik sagt dies wenig aus. Die Quantenmechanik ist aber nach Überzeugung der meisten Physiker nicht experimentell widerlegt. --Franz Scheerer aus Wiesbaden (Diskussion) 10:12, 12. Feb. 2023 (CET)

Natürlich ist die Quantenmechanik nicht widerlegt. Einsteins Konzept des lokalen Realismus jedoch schon. Insofern halte ich den Satz in der Einleitung für falsch: "Damit hatte er einen experimentell überprüfbarer Widerspruch des einsteinschen Konzepts und den Vorhersagen der Quantenmechanik formuliert". Das müsste in meinen Augen heißen: "Damit hatte er einen experimentell überprüfbarer Widerspruch des einsteinschen Konzepts oder der Vorhersagen der Quantenmechanik formuliert." Denn beide können ja nicht falsch sein. --hg6996 (Diskussion) 10:28, 12. Feb. 2023 (CET)
Wieso können nicht beide falsch sein? --Franz Scheerer aus Wiesbaden (Diskussion) 10:35, 12. Feb. 2023 (CET)
Entweder es gibt den beschriebenen Realismus oder nicht. Entweder die Lokalität besteht, oder eben nicht. Nach meinem Kenntnisstand gibt es nur die zwei Möglichkeiten. --hg6996 (Diskussion) 11:42, 12. Feb. 2023 (CET)
"Wurde die Quantenmechanik damit bewiesen?" - Nein. Theorien werden nicht bewiesen. Siehe Duhem-Quine-These. --Hob (Diskussion) 11:31, 12. Feb. 2023 (CET)
Aber die Experimente zeigen, dass die Teilchenpaare tatsächlich verschränkt sind. Wenn die Filter gleich ausgerichtet sind, wird an beiden Orten tatsächlich für jedes Photon die gleiche Polarisation gemessen. Dies lässt sich noch mit verborgenen Variablen erklären. Doch die Messergebnisse bei anderen Filterstellungen widersprechen der Erklärung durch verborgene Eigenschaften der Teilchen. Die Ergebnisse der Messungen können nicht bereits bei der Erzeugung der verschränkten Paare feststehen. Sie sind offenbar echt zufällig und stehen nicht bereits vor der Messung fest. --Franz Scheerer aus Wiesbaden (Diskussion) 09:38, 15. Feb. 2023 (CET)
Ja, die Theorie passt zur Realität und die andere nicht. Aber das nennt man nicht "eine Theorie beweisen". Siehe Duhem-Quine-These. --Hob (Diskussion) 08:29, 17. Feb. 2023 (CET)

Und es gibt sie doch, die spukhafte Fernwirkung, auch schon bei der Gravitation.

Die Gravitationskraft der Sonne auf die Erde ist stets auf den Schwerpunkt der Sonne gerichtet und zwar nicht dort hin, wo sich die Sonne vor etwa 500 Sekunden, der Laufzeit des Lichts, befand, sondern dahin wo die Sonne jeweils aktuell steht. --Franz Scheerer aus Wiesbaden (Diskussion) 18:51, 15. Feb. 2023 (CET)

Wir brauchen gar keine Quantenmechanik dazu. --Franz Scheerer aus Wiesbaden (Diskussion) 18:53, 15. Feb. 2023 (CET)
Die Allgemeine Relativitätstheorie sieht das anders als die Newtonsche Mechanik. Die Realität auch. Die Information über die Position bewegt sich mit Lichtgeschwindigkeit. Siehe Gravitationswelle.
Übrigens dient diese Seite dazu, den Artikel zu verbessern. Sie ist kein Chatroom. --Hob (Diskussion) 08:29, 17. Feb. 2023 (CET)

Braucht man die Bell'sche Ungleichung überhaupt?

Ich frage mich, warum man überhaupt drei Kriterien a, b, c bzw. Größe, Geschlecht, Haarfarbe braucht. Reicht nicht die Tatsache aus, dass die Nicht-Übereinstimmung sin2 mehr als linear wächst?

ARGUMENTATION BEGINN: Gegeben seien zwei verschränkte Photonen, die man in entgegengesetzte Richtung jeweils einen Filter passieren lässt. Wenn die beiden Filter parallel sind, besteht 100 %-ige Übereinstimmung: Wenn Photon (1) seinen Filter passiert, dann auch Photon (2); wenn Photon (1) nicht hindurch kommt, dann Photon (2) auch nicht. Dabei ist egal, in welche Richtung die Filter gedreht sind, Hauptsache sie sind parallel. Die Hypothese der verborgenen Parameter besagt, dass für jedes der Photonen von Anfang an festgelegt ist, wie sie sich bei einer bestimmten Winkelstellung des Filters verhalten werden, und dass diese Festlegungen für beide die gleichen sind.

Verdreht man nun Filter (1) um 6° nach links, so treten Diskrepanzen auf: In sin²(6°) = 1 % der Fälle weicht das Verhalten von Photon (1) vom Verhalten von Photon (2) ab. Der verborgene Parameter in Photon (1) muss also in 1 % der Fälle so eingestellt gewesen sein, dass es sich bei 6°-Drehung des Filters nach links anders verhält. Wenn man statt dessen Filter (2) um 6° nach rechts verdreht, treten ebenfalls in 1 % der Fälle Diskrepanzen auf – diesmal aufgrund der Voreinstellung eines verborgenen Parameters in Photon (2). Wenn man nun sowohl Filter (1) um 6° nach links als auch Filter (2) um 6° nach rechts dreht, sollte die Diskrepanz 2 % betragen, weil sich die Voreinstellungen der Parameter beider Parameter auswirken. Die Diskrepanz müsste sogar etwas geringer sein als die Summe der einzelnen Diskrepanzen, weil sich ja in 0,01 % der Fälle das Verhalten beider Photonen ändert, was wieder eine Übereinstimmung ist.

Die beobachtete Diskrepanz beträgt jedoch nicht 2 % sondern sin²(12°) = 4 %. Dies ist unvereinbar mit der Annahme, dass die beiden Photonen bei ihrer Entstehung verborgene Parameter als „Voreinstellung“ bekommen und sich danach unabhängig voneinander verhalten. ARGUMENTATION ENDE.

Ich persönlich finde das einfacher als a, b, c --- Wassermaus (Diskussion) 23:33, 1. Okt. 2023 (CEST)

Stimmt! Das kommt ohne Indices und fast ohne Formeln aus. Wenn du 30 bzw 60 Grad nimmst, hast du dasselbe Ergebnis. — Reilinger (Diskussion) 10:37, 2. Okt. 2023 (CEST)
Mir ist nicht Du vorschlägst. So, wie ich Deine Argumentation verstehe, nutzt sie implizit auch mehr als zwei Einstellungen, nämlich in Bedingungen (1) und (2) die perfekte Korrelation für zumindest zwei zueinander orthogonale Einstellungen (also insbesondere für h und v) und dann für den verdrehten Winkel. Nur wenn 1 und 2 erfüllt sind, widerspricht die "Diskrepanz" bei 6º dem lokalen Realismus. D.h., die eine Ungleichung wird durch eine Ungleichung und zwei Gleichungen ersetzt. Wenn man also mit (1-3) überprüfen wollte, ob die Messergebnisse sich mit lokalen verborgenen Variablen erklären lassen, müsste man zunächst (1)+(2) nachweisen, aber perfekte Korrelationen wird man nie finden. Dann ist eine quantitative Formulierung nötig, dass bei kleinen Abweichungen von (1)+(2) eine hinreichend grosse Diskrepanz bei 6º immer noch inkompatibel ist mit lokalem Realismus. Die drei Richtungen im Beispiel wurden mE gewählt weil die klassische Logik (eine Bedingung weglassen kann die Wahrscheinlichkeit für das Ereignis bloss vergrössern) leicht nachvollziehbar ist. Mann kann auch mit nur zwei Einstellungen arbeiten (wie in der CHSH-Ungleichung), aber die Ableitung der Ungleichung ist dann etwas komplizierter. Ausserdem nutzte, soweit ich weiss, der erste Beweis von Bell drei Einstellungen und der Artikel folgt diesen historischen Beispiel ("Die ursprüngliche Überlegung..."). Aber das muss man natürlich nicht so machen. Was mMn aber Bestand haben sollte, ist, eine Gleichung abzuleiten, deren Verletzung im Widerspruch mit allen denkbaren LHV-Theorien steht. --Qcomp (Diskussion) 12:18, 2. Okt. 2023 (CEST)
Ja gut, ich hätte in der Tat statt von 6° und 12° besser von 30° und 60° sprechen sollen. Zum einen, weil es in den anderen Ausführungen auch steht, und zum anderen, wie du sagst, weil man "perfekte Korrelation nie finden" wird -- ein valides, praktisches Argument. Aber ansonsten? Was wollen wir: möglichst nahe bei Bells Paper bleiben oder es so schreiben, dass es für den Laien verständlich wird und dass der Fachmann es im Skilift, ohne Papier und Bleistift seinem Kumpel erklären kann? Am besten doch beides.
Mit 30° ist die Argumentationskette wie folgt:
1) wenn beide Filter parallel sind, verhalten sich die Photonen zu 100 % gleich. Wenn Photon (1) seinen Filter passiert, dann auch Photon (2); wenn Photon (1) nicht hindurch kommt, dann Photon (2) auch nicht. Dies ist experimentell nachweisbar. Es ist eine Voraussage der QM. Man kann es genausogut durch lokale verborgene Parameter erklären.
2) Aussage 1) gilt immer, egal wie man die Filter dreht, Hauptsache sie sind parallel. Das heißt: die verborgenen Parameter müssen für jeden Filterwinkel eine Voraussage machen, und zwar stets für beide Photonen die gleichen.
3) Verdreht man Filter (1) um 30° nach links, so treten in sin2(30°) = 14 der Fälle Unterschiede beim Verhalten von Photon (1) vom Verhalten von Photon (2) auf. Auch das kann man sowohl durch QM als auch durch lokale Parameter erklären. (Man beachte: ich brauche hier nicht v und h einzeln zu nennen - daher wird das Bild laientauglicher; meine o.g. Bemerkung, daran sei der lokale Parameter in Photon 1 schuld, war falsch, es geht um beide parameter.).
4) Das gleiche gilt, wenn man statt dessen Filter (2) um 30° nach rechts verdreht.
5) Wenn man sowohl Filter (1) um 30° nach links als auch Filter (2) um 30° nach rechts dreht, sollten sich beide Diskrepanzen addieren. Die Diskrepanz müsste sogar geringer sein als 14+14=12 (nämlich 716), weil sich in 116 der Fälle das Verhalten beider Photonen zugleich ändert, was wieder als Übereinstimmung zählt. Die QM aber sagt: die Diskrepanz beträgt 34. Und das ist experimentell unterscheidbar.
Ich meine, das ist leichter verständlich und leichter erklärbar (sogar ohne Papier und Bleistift) als so was mit , , usw usw. Ob man das noch als Bellsche Ungleichung bezeichnen kann oder als "von der Bellschen Ungleichung inspiriert", ist vielleicht Ansichtssache.
-- Wassermaus (Diskussion) 13:03, 2. Okt. 2023 (CEST)
Es ging mir eigentlich nicht um die Grösse des Winkels, sondern um den Unterschied "eine Ungleichung" vs. "mehrere Gleichungen". Ich hab prinzipiell nichts dagegen, eine andere als die "historische" Ableitung zu verwenden, aber ich finde es wichtig, am Ende eine Ungleichung zu haben, deren Verletzung schon zum Widerspruch mit dem lokalen Realismus führt. Aber Deine Argumentation braucht mehr, nämlich den Nachweis all der in (1-5) beschriebenen Resultate, die zudem alle Gleichungen sind. Das macht es schwieriger, einen praktisch sichtbaren und quantifizierbaren Widerspruch zwischen QM und LHV zu konstruieren.
  • Das Auftreten von Gleichungen in allen 5 Bedingungen führt dazu, dass in der Praxis auch die QM-Vorhersagen "verletzt" (bzw nur im Rahmen der Messgenauigkeit (inkl Imperfektionen von Zustandspräparation und Detektoren) erfüllt) werden und dann, um LHV mit einem bestimmten P-Wert auszuschliessen, nachgerechnet werden müsste, wie wahrscheinlich das erhaltene Resultat (für (1-5)) für das optimale LHV-Modell wäre (vgl. z.B. arxiv:1510.07233).
  • Ungleichungen (mit einer endlichen Verletzung durch die QM, wie ) sind robust unter Imperfektionen, da S stetig vom verwendeten Zustand und den Messoperatoren abhängt, Gleichungen sind es nicht.
  • In der Regel würde man sagen, wir machen ein Experiment und zeigen (1) und dann wissen wir, dass das gilt und jetzt überprüfen wir (2) usw. Aber das ist nicht, was man im Kontext von Bell-Ungleichungen machen möchte, weil man ja alle denkbaren LHV-Theorien ausschliessen will und daher auch völlig hypothetische Einflüsse, die z.B. die Detektoreinstellung bei "A" auf die Messresultate bei "B" haben könnte (wenn sie nicht raumartig getrennt sind in dem Sinn, dass keine Information mehr rechtzeitig vom einen Photon zum anderen gelangen könnte, um die LHV entsprechend "anzupassen"; Stichwort locality loophole) und man müsste also, ein Experiment machen, in dem man die für (1)-(5) nötigen Einstellungen zufällig und raumartig voneinander separiert justieren kann, um den Ansprüchen der Kollegen zu genügen, die "pro lokaler Realismus" argumentier(t)en. (Wobei das sich vielleicht sogar auf vier Einstellunge enidampfen lässt: manche Annahmen in (1-5) sind redundant (ich muss nicht perfekte Korrelation für alle Winkel nachweisen, um LHV zu verletzen), wobei mir nicht klar ist, ob die vier Einstellungen (H,H), (H+30,H), (H,H-30), und (H+30,H-30) schon genügen).
  • Deine Argumentation ist nah an einer der ersten Formulierungen von Bell: On the Einstein Podolsky Rosen paradox. In: Physics. Band 1, 1964, S. 195, doi:10.1103/PhysicsPhysiqueFizika.1.195., der auch eine "Diskrepanz-Ungleichung" bekommt, die aber auch auf der Annahme von drei Gleichungen basiert. Meines Wissens wurde diese Ungleichung in der Praxis durch spätere Ungleichungen (wie CHSH) wegen der besseren Überprüfbarkeit letztere abgelöst.
  • Ich finde den einen nicht-trivialen Schritt in den beiden Argumentationen (nämlich für drei dichotome Observable -wobei die erste Gleichung annimmt, dass alle drei Werte gleichzeitig bestimmt sind und die zweite, dass das Weglassen von Bedingungen die Wahrscheinlichkeit allenfalls erhöht) umseitig bzw Deinen Schritt (5)) beide ähnlich einleuchtend. Daher würde ich Stand jetzt die derzeitige Formulierung, die eine ohne Nebenbedingung überprüfbare Bellungleichung ableitet, vorziehen.
  • Man kann auch (wie es historisch war) in zwei Schritten vorgehen: (1) zeigen, dass die Vorhersagen der idealen QM nicht mit LHV kompatibel sind (Dein Zugang) und dann, dass sich robuste Ungleichungen finden lassen, die eine Separation QM <-> LHV erlauben, da sie von der QM um einen Betrag der Grössenordnung 1 verletzt werden. --Qcomp (Diskussion) 15:19, 2. Okt. 2023 (CEST)
Den letztgenannten Punkt finde ich super (die anderen Punkte sind natürlich auch gut). Das Thema hat mich die Tage nicht losgelassen, und heute auf einer langen Bahnfahrt habe ich noch mal überlegt, wie das noch gleich war mit der spukhaften Fernwirkung und den widerlegten Lokalen Parametern. Das eine Beispiel (30 Grad links → 25 % Diskrepanz, 30 Grad rechts → 25 % Diskrepanz, beide Drehungen → 75 % Diskrepanz) konnte ich sofort memorieren und hätte es auch meinem (nicht vorhandenen) Sitznachbarn erklären können. Aber das mit N h v a b c ... im Kopf oder mit Zettel nachzuvollziehen ohne irgendwo nachzulesen, das war mir zu komplex. -- Reilinger (Diskussion) 20:18, 5. Okt. 2023 (CEST)

Ungrammatischer Satz?

Unter "Realismus und Lokalität" steht unter 2.

Eine physikalische Theorie ist nicht lokal, im Sinne der speziellen Relativitätstheorie, wenn in raumartiger Relation die Messergebnisse an zwei Teilchen korreliert sind, eine dem Zufall widersprechende Beziehung zeigen, ...

Der Gliedsatz ", wenn ..." endet mit "korreliert sind,". Was ist aber das Segment "eine ... zeigen"? Ein weiterer Gliedsatz (dann fehlt die Konjunktion oder das Relativpronomen)? Ein gleichgeordneter Satzteil mit einem fehlenden "das heißt"? Der Satz ist m.E. grammatikalisch falsch und damit unverständlich - aber vielleicht blicke ich nur nicht durch ... --Haraldmmueller (Diskussion) 08:09, 30. Nov. 2023 (CET)

Ja, so sehe ich das auch. Evtl. ist gemeint: "Eine physikalische Theorie ist nicht lokal, im Sinne der speziellen Relativitätstheorie, wenn in raumartiger Relation die Messergebnisse, die an zwei Teilchen korreliert sind, eine dem Zufall widersprechende Beziehung zeigen, ohne dass dies mit objektiv vorliegenden Eigenschaften der Teilchen erklärt werden könnte."
--Zulu55 (Diskussion) 08:52, 30. Nov. 2023 (CET)
Wär zwar grammatisch ok - aber inhaltlich kann ich "Messergebnisse, die an zwei Teilchen korreliert sind" auch nur mit wackeliger (Um-)Deutung verstehen. Ich probier: Eine physikalische Theorie ist nicht lokal im Sinne der speziellen Relativitätstheorie, wenn Messergebnisse an zwei in raumartiger Relation voneinander entfernten Teilchen eine dem Zufall widersprechende Beziehung zeigen, ohne dass dies mit objektiv vorliegenden Eigenschaften der Teilchen erklärt werden könnte. Ja, da hab ich nun mehr geändert = ist nun Frage der physikalischen Korrektheit; aber so, denke ich, hab ich's gelernt. --Haraldmmueller (Diskussion) 11:58, 1. Dez. 2023 (CET)